Anda di halaman 1dari 76

SOAL PENGANTAR 1 (A) denotasi

SEMANTIK I (B) leksikal


(C) gramatikal
1. Kami menantikan angin baik (D) kiasan
untuk melancarkan agresi menentang (E) ganda
mereka. Makna kata angin dalam
kalimat di atas ialah .... 7. Tak peduli kondisi bangsa dan
(A) Kesempatan negara kita yang sungguh kian
(B) Cuaca memprihatinkan, mereka justru kian
(C) Keadaan manja dan jemawa. Ibarat gergasi,
(D) Kabar mereka tak henti-hentinya
(E) Suasana mengajukan aneka macam tuntutan
akan tunjangan, kelengkapan,
2. Ungkapan di bawah ini gratifikasi, fasilitas, apreseasi, dan
mengandung makna idiomatis, privilise tanpa sedikit pun
kecuali .. menimbang betapa rendahnya laku,
(A)Bertangan tingin kualitas, dan bakti mereka. Makna
(B) Kepala dingin kata privilise pada kalimat di atas
(C)Berhati dingin adalah ...
(D) Berdarah dingin (A)Hak istimewa
(E) Terasa dingin (B) Tanda jasa
(C) Kekebalan hukum
3. Dari kelima ungkapan diberikut ini, (D) Bantuan kepartaian
yang mengandung makna cepat (E) Kunjungan kerja
tersinggung ialah ....
(A) Luka hati Setiap suku memiliki ritual tersendiri,
(B) Patah hati mulai dari ritual yang biasa hingga
(C) Sempit hati (8) ekstrem , untuk menunjukkan rasa
(D) Berat hati duka akibat kehilangan anggota
(E) Kecil hati keluarganya. Bagi suku Dani yang
bermukim di Papua ,
4. Sinonim dari kata realisasi ialah .... kebersamaan sangat penting. Oleh
karena itu, saat kehilangan anggota
(A) Pergerakan keluarga, mereka akan segera
(B) Perwujudan memotong ruas jarinya. Tradisi itu
(C) Penciptaan dikenal sebagai Iki Palek.
(D) Pembuatan 8. (A) aneh
(E)Pengadaan (B) lazim
(C) ganjil
5. Makna diberikut yang tidak (D) luar biasa
mengandung makna idiomatis ialah ... (E)TIDAK PERLU DIPERBAIKI
(A) Ringan tangan
(B) Buah hati 9. Saat ini kami sedang
(C) Meja hijau mempersiapkan berkas-berkas
(D) Rambut merah akreditasi karena minggu depan akan
(E) Jago merah dilakukan kunjungan oleh para
pengawas. Kata yang tepat untuk
6. Orang itu ternyata panjang tangan. menggantikan kata akreditasi
Ungkapan yang bergaris bawah di adalah…
atas mengandung makna ....
(A) Penilaian 12. Kalimat berikut yang
(B) Laporan mengandung makna denotasi adalah
(C) Pertanggungjawaban ….
(D)Dokumen
(E) Pengakuan (A)Jumlah tunakarya di kotaku
semakin meningkat saat pandemi
10. Antonim kata khusus adalah... Covid-19.
(A) milineal (B)Bini Pak Andi merupakan seorang
(B) metropolitan guru yang sangat sabar.
(C) minoritas (C)Pencuri itu dipukuli warga hingga
(D)global mampus.
(E) genre (D)Aminah membantu ibunya
mencuci baju.
Beberapa rezim pemerintahan yang (E) A, B, C, dan D benar
berkuasa memang memberi perhatian
pada sektor pendidikan, tetapi 13. Dalam kasus malapraktik dokter,
perhatian tersebut kurang memadai. sebenarnya ada duapelanggaran,
Pemerintah terjepit keharusan yakni pelanggaran profesi dan
membayar cicilan bunga maupun pelanggaran hukum. Namun, selama
utang pokok pinjaman yang makin ini dalam setiap kasus malapraktik,
lama makin membesar hingga dokter selalu di pihak yang benar.
melampaui tingkat batas yang Makna istilah malapraktik dalam
ditoleransi. Batas toleransi yang paragraf tersebut adalah....
dimaksud adalah antara 20 persen (A) izin praktik
hingga 30 persen. Di lain pihak,  (B) waktu praktik
sektor-sektor lain seperti infrastruktur (C) kemandirian praktik
juga memerlukan perhatian. (D) kesalahan praktik
ironisnya, anggaran yang sudah (E) penyimpangan praktik
menipis itu masih juga dikorupsi
hingga yang terkadang hanya cukup 14. Menurut sebuah situs di internet,
untuk kegiatan rutin. Dalam kondisi gunung api di atas permukaan laut
seperti ini, pemerintah tidak memiliki masih tergolong stratovulakno.
dana khusus yang memadai buat anak Makna kata situs pada kalimat di atas
didik yang berbakat dan cemerlang adalah …..
serta bermoral baik. Berbeda dengan (A) Daerah temuan benda-benda
kerajaan Malaysia yang secara pubakala
khusus… anggaran untuk anak didik (B) Laman sebuah komputer
berprestasi. (C) Mesin pencari informasi
(D) Tempat untuk lambang suatu
 11. Kata yang tepat untuk inskripsi
melengkapi kalimat terakhir adalah... (E) Alamat
(A) menyediakan
(B) bersedia
(C) merespons
(D) berpedoman
(E) membuktikan
15. “Terdapat pola menyerupai busur 19. Hiponim kata burung tertera
mulai dari Cianjur-Jatiluhur di dalam kalimat-kalimat di bawah ini,
sebelah barat laut, Subang di bagian kecuali ....
utara dan timur laut, serta Gunung (A)Merak mempunyai bulu yang
Tampomas-Majalengka di sebelah sangat indah.
timur laut.” Makna istilah yang (B)Itik liar termasuk binatang yang
bercetak tebal dalam kalimat tersebut dilindungi.
adalah... (C)Merpati dipakai untuk
(A) Berbentuk bulat mengantarkan pesan.
(B) Berbentuk setengah lingkaran (D)Perkutut kakek suaranya sangat
(C) Berbentuk lonjong bagus.
(D) Berbentuk seperti panah (E)Populasi kakaktua putih semakin
(E) Berbentuk cembung berkurang.

16. Dari contoh di bawah ini mana 20. Kucing makan tikus mati di
yang termasuk antonim mutlak? dapur. Kalimat tersebut menimbulkan
(A) Aku – Saya kesalahan berbahasa yang berupa ....
(B) Tinggi – Rendah (A) redundansi
(C) Tinggi – Pendek (B) ketaksaan
(D) Saya – Cantik (C) ejaan
(E) Saya – Ganteng (D) tanda baca
(E) kerancuan
17.Melihat dekat antara Pak Jono
dengan Putra, Tono sangsi Pak Jono 21. X hari ini tidak memimpin rapat
akan memberikan sanksi kepada karena sakit. Menurut dokter, X
Putra yang sudah memecahkan kaca perlu istirahat yang cukup. Sementara
jendela ruang guru. Dari kutipan saya yang menggantikan X. Tulat
tersebut terdapat kata 'sangsi' dan beliau X sudah masuk kembali.
'sanksi' yang memiliki hubungan? Makna kata tulat adalah ...
(A) hari sesudah lusa
(A) Homograf (B) dua hari sesudah hari ini
(B) Homonim (C) pesan sebelum sakit
(C) Polisemi (D) pelimpahan wewenang
(D) Hiponim (E) minggu depan
(E) Homofon

18. Penduduk desa binaan 22. Para siswa melakukan registrasi


RidwanKamil mulai membajak hari ini. Kata registrasi pada kaliamt
sawah. Mereka akan menanam di atas adalah....
padi karena musim huja sudah (A)Mendaftar
hadir. Penggunaan kata yang tidak (B) Mendaftar ulang
tepat pada paragraf di atas (C) Proses mendaftar ulang
adalah .... (D) Proses pendaftaran
(A) Binaan (E) Pendaftaran
(B) Membajak
(C) Musim
(D) Hadir
(E) Hujan
23. Kalimat berikut yang termasuk
makna leksikal adalah...
(A) Salah satu finalis putri daerah itu di dunia ini dapat dikelompokkan atas
hanya jual tampang saja di persamaan dan perbedaannya.
panggung
(B) Penjualan daging sapi menurun Ragam makna antonim yang terdapat
drastis pada teks di atas adalah ….
(C) Dia berpendapat bahwa Buka A. hidup – mati / vertebrata –
Lapak salah satu situs jual invertebrata / persamaan –
beli online terpercaya di perbedaan
Indonesia B. hidup – mati / diklasifikasikan –
(D) Mahasiswa yang mendapat nilai dikelompokkan / bergerak –
IPK tertinggi di universitas tidak dapat bergerak
ternama itu jual mahal ketika C. binatang – tumbuh-tumbuhan /
diajak bekerja sama dengan salah pertama – kedua / persamaan –
satu perusahaan terbesar di Asia perbedaan
(E) Penelitian eksklusif terbaru oleh D. benda di dunia ini / menjadi
Unicef menemukan fakta bahwa binatang dan tumbuhan /
banyak gadis di Kenya dipaksa makhluk mati
jual diri untuk E. binatang – tumbuhan / vertebrata
mendapatkanproduk sanitasi, – invertebrata / satu tempat –
yang disebabkan tempat lain.
situasi kemiskinan .

24. Kata panas bermakna leksikal


pada kalimat di bawah ini adalah....
(A) Susana makin panas ketika
pemimpin baru itu datang
menghadiri rapat tertutup
(B) Akhirnya, lelaki itu duduk di
kursi panas
(C) Di belahan utara bumi, musim
panas dimulai sekitar tanggal 21
Juni hingga 23 September
(D) Dia memanaskan teh tawar itu
(E) Wol adalah salah satu bahan
untuk membuat baju panas

25. Semua benda di dunia ini dapat


diklasifikasikan menjadi benda hidup
dan benda mati.Benda hidup dapat
dikelompokkan lagi menjadi binatang
dan tumbuh-tumbuhan.Binatang
dapat dibagi menjadi vertebrata dan
invertebrata.
Yang pertama sering disebut
makhluk hidup dan kedua sering
disebut makhluk mati.Tumbuh-
tumbuhan tidak dapat bergerak dari
satu tempat ke tempat lain sedangkan
binatang dapat bergerak bebas.Benda
SOAL PENGANTAR 2
(E) Peyorasi
SEMANTIK II & III
6. Pesawat terbang termasuk alat
1. Hubungan kemaknaan antara transportasi modern.
sebuah kata atau satuan bahasa yang
lainya lagi disebut.... Kata modern berantonim dengan..
(A) makna leksikal
(B) makna denotasi (A) Regional
(C) makna gramatikal (B) Nasional
(D) makna konotasi (C) Internasional
(E) relasi makna (D) Tradisional
(E) Konvesional
2. Menjadi orang kaya jangan
melupakan orang miskin. Kalimat 7. Kalimat berikut yang
tersebut merupakan antonimi yang menggunakan kata bermakna
bersifat .... peyorasi adalah..
(A) mutlak
(B) kutub (A) Pejabat itu sering menerima
(C)relasional amplop dari bawahannya.
(D) hierarkial (B) Pemimpin gerombolan di
(E)majemuk Kota A sudah ditangkap.
(C) Peran wanita sangat besar di
3. Contoh kata yang bukan polisemi masa sekarang ini.
adalah (D) Kata-katanya selalu lemah-
(A) buah lembut kepada siapa saja.
(B) raja (E) Tersangka kasus korupsi
(C) kepala dibawa ke lembaga
(D) kursi permasyarakatan
(E) buku

4. Contoh kata berhomofon 8.Untuk membangun tanggul


adalah .... penahan banjir, pemerintah akan
(A) massa dan masa merelokasi warga yang tinggal di
(B) keset dan keset bantaran sungai. Yang dimaksud
(C) bulan dan bulan dengan bantaran dalam kalimat di
(D) kepala dan kepala atas adalah ...
(E) bunga dan mawar (A) bagian hilir.
(B) bagian hulu.
(C) bagian sisi kanan dan kiri.
5. Kata-katamu sungguh pedas (D) bagian yang tidak dialiri air.
untuk didengar. (E) bagian lereng tebing.

Senyumannya manis sekali. 9. Kata yang mengalami perubahan


makna karena asosiasi terdapat
Kalimat diatas termasuk dalam dalam kaiimat ...
perubahan makna kata? (A) Persentase sarjana yang belum
(A) Sinestesia mendapatkan pekerjaan semakin
(B) Generalisasi bertambah.
(C) Asosiasi
(D) Spesialisasi
(B) Para pegiat perempuan semakin 12. Para maniak sepak bola dibuat
berani memperjuangkan hak- lupa daratan oleh permainan bola
haknya. ajaib yang telah menjadi salah satu
(C) Karena kesantunannya, oleh ikon terbesar budaya olahraga
temannya dia disebut sebagai tontonan manusia abad XX−XXI.
pendeta. Makna ungkapan lupa daratan
(D) Sesekali dia melontarkan dalam kalimat di atas adalah ...
kritikan pedas terhadap kebijakan (A) lupa diri.
atasannya yang dianggapnya keliru. (B) tidak peduli pada hal yang lain.
(E) Pada waktu melaksanakan tugas (C) bertindak sesuai dengan harga
jurnalistiknya, dia selalu berusaha diri.
menolak pemberian amplop dalam (D)terlalu asyik.
setiap peristiwa yang diliputnya (E)bersorak-sorak

10. . ....., ...... bahwa air yang 13. Taman Wisata Candi Borobudur
dikonsumsi masyarakat di daerah itu akan dijadikansebagai lembaga ...
mengandung sejumlah zat gajah dan untukmemperbanyak ...
berbahaya akibat pencemaran. gajah, tahun ini, Taman
Bagian kalimat yang tepat untuk WisataCandi Borobudur akan
melengkapi kalimat tersebut adalah menambah dua ekor gajah lagi,
(A) Setelah penelitian, diketahui jantan dan betina.
(B) Setelah dilakukan penelitian, Kata yang tepat untuk mengisi
mengetahui kalimat rumpang tersebut adalah ...
(C) Setelah melakukan penelitian, (A) preservasi, habitat.
mengetahui (B)rehabilitasi, komunitas.
D. Setelah meneliti, diketahui (C)pelestarian, spesies.
E. Setelah diteliti, diketahui (D)konservasi,populasi.
(E)penelitian, habitat.
11. Hujan yang turun empat hari
terakhir di wilayah Gunung Kidul 14.Negara Indonesia mempunyai
mampu ........ 41.752 hektare lahan bahasa......, yakni bahasa Indonesia.
padi yang semula sudah layu. Bahasa Indonesia
Bahkan seluruh tanaman pangan merupakan........bangsa yang sangat
dapat ...... kembali. Di wilayah penting kedudukannya. Kata
utara, seperti Kecamatan Semin, bentukan baku yang tepat untuk
Ngawen, dan Nglipar sudah ....... mengisi bagian yang rumpang
panen jagung dan kedelai. dalam kalimat di atas adalah........
Bentuk kata yang tepat untuk (A)pemersatu,persatuan
mengisi titik-titik di atas adalah (B) persatuan, pemersatu
(A) menghancurkan, merusak, (C) persatuan, persatuan
berakhir (D) persatuan, mempersatukan
(B) menyelamatkan, menghijaukan, (E) pemersatu, mempersatukan
dilakukan
(C) membanjiri, ditanami, dilakukan 15. Seorang ahli hukum tata negara
(D) menggilas, menguning, menyebut hal inisebagai bentuk
disibukan "kudeta redaksional".Makna kata
(E) menyelamatkan, dirusak, kudeta pada kalimat di atas
digalakkan adalah ...
(A) penggalangan kekuasaan.
(B) penggulingan kekuasaan.
(C)pengambilalihan kekuasaan. (E) Tidak adil bila yang kaya raya
(D)perebutan kekuasaan dengan tidak dikenakan pajak.
paksa.
(E)pengakuan kedaulatan
19. Berikut ini adalah antonim
16. Kata bapak yang mengalami majemuk dari kata berdiri ….
perluasan makna terdapat dalam (A) Berlari, melompat, jongkok,
kalimat… tidur-tiduran
(A) Di mana alamat rumah Bapak (B) Duduk, tidur, tiarap, jongkok
yang memberi ceramah tadi? (C) Berjinjit, melompat, tengadah,
(B) Kepala sekolah mengumpulkan menguap
bapak-bapak dari siswa kelas IX. (D) Terlentang, terjatuh, duduk,
(C) Bapak saya bekerja sebagai tidur
dokter di rumah sakit ini. (E) Duduk, berlari, melompat,
(D) Itukah bapak si murid baru di tengadah
kelas kita?
(E)Bapaknya menerima penghargaan 20. Kalimat yang mengandung kata
Kalpataru berpolisemi …
A. Setiap bulannya adik mendapat
bunga dari depositonya.
B. Kami semua berjalan kaki sampai
17. Setelah keduanya di atas ring mereka ke kaki gunung.
berhadapan dengan sikap C. Kakinya senang memakai baju
menantang. hijau karena memang ia
Makna gramatikal imbuhan ber-an begitulah seragamnya.
pada kata berhadapan dalam kalimat D. Orang itu dibebastugaskan karena
di atas adalah … melanggar peraturan yang utama.
E. Para tunawisma datang setiap
(A) sebagai pemanis Jumat ke pasturan itu.
(B) menegaskan arti
(C) melakukan pekerjaan
(D) menyatakan saling
(E) mempunyai

18. Kalimat di bawah ini yang tidak


menggunakan kata berantonim
adalah …
(A) Tuhan tidak melihat tinggi
rendahnya derajat mereka di
dunia.
(B) Siang malam kedua orang itu
menunggu anaknya yang belum
pulang.
(C) Jangankan yang muda yang tua
pun ikut menyaksikannya.
(D) Bunga pinjaman ditentukan
berdasarkan besar kecilnya
pinjaman.
SOAL PENGANTAR 3 4. Buah-buahan yang dibeli Inul di Pasar
Baru masih tampak segar. Buah-buahan
REDUPLIKASI dalamkalimat tersebut tergolong kata
ulang….
(A) berubah bunyi
1.Karena kehujanan sepulang les (B) sebagian
kemarin sore, sekarang Melati batuk- (C) murni.
batuk dan flu.Makna kata ulang berikut (D) berimbuhan
yang semakna dengan kata ulang batuk- (E)semu
batuk pada kalimat di atas adalah….
(A) Arya berjalan-jalan sore itu. 5. Kata ulang yang menyatakan makna
(B) Dilihatnya anak-anak sedang latihan saling adalah…
capoera di sisi timur bangsal. (A) Salah satu ciri khas bangsa
(C) Ada yang menggelitik hatinya di Indonesia dalam bermasyarakat adalah
antara rerimbunan kembang puring. tolong-menolong.
(D) Diamatinya lekat-lekat tanpa (B) Anak itu melempar-lempar
berkedip sedetik pun. mainannya.
(E) Ternyata, Eren sedang mencari laba- (C) Paman tidur-tiduran di atas sofa.
laba bersama Azhar. (D) Berbulan-bulan kakaknya tidak
pulangdari perantauan.
2. Pasutri itu menimang-nimang anaknya (E) Pohon-pohon di tepi jalan sangat
sambil bernyanyi. menganggu pada saat hujan deras tiba.
Proses pembentukan kata ulang pada
kalimat tersebut sama dengan kalimat…. 6.. Pengulangan kata yang menyatakan
(A) Ibu itu bolak-balik saja di koridor paling (SUPERLATIF) terdapat pada
sedari tadi. kalimat ….
(B) Intan menyobek-nyobek kertas (A) Pemusik-pemusik di era
ulangannya yang mengecewakan di Indonesia baru inibanyak
depan kawaan-kawannya. bermunculan.
(C)Foto-foto itu dicetaknya di studio (B) Mereka selalu berupaya untuk
Wahid. menciptakan lagu dengan sebaik-
(D)Yulita menulis-nulisi buku Esti tanpa baiknya.
sadar. (C) Penyanyi-penyanyi muda dan
(E)Kami-kami ini sering menjadi ceria selalutampil dengan gaya
sasaran ejekan teman sekelas. yang berbeda.
(D) Mereka kenal-mengenal sebelum
3. Yang merupakan kata ulang semu memulaipertandingan.
adalah…. (E) Pemuda-pemudi mengikuti
(A) buku-buku peringatan 17 Agustus di
(B) kupu-kupu Lapangan Benteng tadi pagi.
(C) pontang-panting
(D) baris-berbaris .
(E) tanam-tanaman
7. Adik saya memiliki mobil-mobilan (A) Tamu itu tukar-menukar
antik yang tidak dimiliki oleh teman cenderamata.
sebayanya. Makna kata yang bercetak (B) Kakak saya senang masak-
miring tersebut adalah…. masakan.
(A) Superlatif (C) Jangan berkejar-kejaran nanti
(B) Frekuentatif jatuh.
(C) Imitasi (D) Kedua orang yang berkelahi itu
(D) Kuantitas akhirnya bersalam-salaman.
(E) Kualitas (E) Dari tadi sama mengetuk-ketuk
pintu rumahnya.
8. Penggunaan kata ulang dalam kalimat
berikut yang tepat adalah …. 11. Kata yang sepola denga gerak-gerik
adalah....
(A) Gadis-gadis di SMU putri itu (A) Teka-teki
sangat cantik. (B) Utak-atik
(B) Rumah di perumnas itu kecil- (C) Ualng-alik
kecil. (D) Pontang-panting
(C) Teman sekolah saya rajin-rajin (E) Pernak-pernik
sekali.
(D) Bunga di taman itu sangat indah- 12. Kata sepola dengan sayur mayur
indah. adalah....
(E) Banyak taman-taman bunga di (A) Hingar-bingar
pegunungan itu luas-luas. (B) Gelap-gulita
(C) Serta-merta
(D) Ramah-tamah
9. Penggunaan kata ulang yang tepat (E) Tunggang-langgang
terdapat pada kalimat …
13. Kata sepola dengan kata sesuatu
(A) Kedua petinju itu saling pukul- adalah..
memukul untuk memperebutkan (A) Tetangga
kejuaraannya. (B) Pepatah
(B) Para orang-orang tua diharapkan (C) Rerata
selalu membimbing anak- (D) Gegabah
anaknya. (E) Pegunungan
(C) Di pusat pertokoan terdapat
anak-anak sekolah yang hanya 14. Pasangan kata yang bukan
melihat-lihat keramaian. merupakan kata ulang adalah....
(D) Perjalanan-perjalanan itu sedang (A) Pertama-tama
diperbaiki oleh pihak (B) Lauk-pauk
pemerintah. (C) Paru-paru
(E) Mereka saling berangkul- (D) Muda-mudi
rangkulan ketika sekolahnya (E) Gerak-gerik
dinyatakan sebagai pemenang,

10. Kata ulang kata kerja yang bermakna


pekerjaan dilakukan berulang-ulang
terdapat dalam kalimat …
15. Adik sedang menyampuli buku
sejarah di kamar depan.

Makna afiks me-i pada kata


menyaampuli di atas sama dengan
makna me-i dalam kalimat

(A) Di belakang rumah kakak sedang


membului ayam.
(B) Kemarin sore saya menemani ibu
ke toko.
(C) Anak-anak suka melempari
mangga tetangga.
(D) Dokter mengobati patiennya tiap
hari.
(E) Guru memarahi murid yang
nakal.
SOAL PENGANTAR 4 5. Deretan gabungan kata yang
mempunyai yang sama dengan
KOMPOSITUM sudut pandang adalah .....
(KATA MAJEMUK) (A) Jawaban soal
(B) Cetak ulang
(C) Segitiga
1. Di bawah ini merupakan kata (D) Lompat jauh
majemuk, kecuali.... (E) Hasil tes

(A) Sepak bola 6.Dilihat dari unsur-unsur


(B) Rumah saki pembentuknya, kata majemuk daya
(C) Kolam renang juang sepola dengan kata majemuk ...
(D) Ramah tamah (A) Simpan pinjam
(E) Kereta api (B) Simpang siur
(C) Lomba lari
2. Kata majemuk senyawa di bawah (D) Kamar tunggu
ini, kecuali... (E) Meja makan
(A) Dukacita
(B) Matahari
(C) Kacamata 7. Gabungan kata majemuk yang
(D) Saputangan memiliki dua inti adalah...
(E) harga diri (A) pecah belah
(B) rumah sakit
3. Orang tua siswa dipanggil (C) rumah makan
Kepsek. Kata orangtua sama dengan (D) .kamar mandi
konteks kalimat di bawah ini.... (E) kapal laut
(A) Dia membawa baju hangat
(B) Saya menyeberangkan orang 8. Kata-kata di bawah ini adalah
tua itu kata majemuk, kecuali...
(C) Sayah meminum kopi panas (A) ayam itik
buatan ibu (B) kereta api
(D) Kertas dipindahkan ke meja (C) rumah makan
hijau sebelah kanan (D) sapu lidi
(E) Kami mencoba menjawab (E) ramah tamah
soal rumit itu

4. Semua bentuk komposisi kata 9.Gabungan kata yang setipe dengan


di bawah ini sama polanya, meja plastik terdapat pada...
kecuali... (A) kursi panjang
(A) Burung terbang (B) dalam mimpi
(B) Rumah warga (C) jeruk manis
(C) Keras hati (D) tepung beras
(D) Toko pakaian (E) kucing hitam
(E) Warung pojok
14. Kalimat yang mengandung
10. Kata majemuk berikut gabungan kata yang berpola sama
yang menunjukkan makna sangat, dengan rendah hati terdapat dalam
kecuali… kalimat…
(A)Susah payah (A)Tindakan yang luar biasa harus
(B) Gelap gulita diambil untuk memulihkan
(C)Ratap tangis keamanan di daerah ini.
(D)Hutan rimba (B) Sudah lama orang tuanya
(E) Kampung halaman berdagang barang pecah-belah.
(C) Sanak saudaranya selalu
11. Pola penggabungan kata yang mengirim berita mengenai keadaan
sama dengan pola biru laut negara leluhurnya.
adalah… (D)Buruh perusahaan itu berunjuk
(A)Mobil baru rasa menuntut kenaikan upah.
(B) Hitam sekali (E) Orang itu memang tampan,
(C) Kamar tidur tetapi ia panjang tangan.
(D)Hitam manis
(E) Merah darah 15. Kalimat yang mengandung
konjungsi korelatif adalah…
12. Ditinjau dari tata bentuk, maka (A)Keadaan memang sedang susah,
gabungan kata yang benar adalah… tetapi kita jangan menyerah.
(A) Perserikatan Bangsa-Bangsa (B) Penumpang dilarang merokok
(B) Perhimpunan Pelajar-Pelajar ataupun meludah dalam bus.
(C) Apakah Anda setuju atau tidak,
Indonesia
kami akan jalan terus.
(C) Metode-metode ilmiah (D) Kami mengundang ketua dan
(D) Suatu gagasan-gagasan sekretaris.
(E) Seperangkat alat-alat salat (E) Saya mau pergi kalau pekerjaan
rumah saya selesai.

13.Kelompok kata yang


memperlihatkan adanya pertalian
yang rapat terdapat pada
konstruksi…
(A) baju tebal
(B) teman saya
(C) cuci gudang
(D) penyajian materi
(E) berjalan cepat
SOAL PENGANTAR 5 (B) Dengan terpaksa, dia
membukakan pintu untuk pengamen
MORFOLOGI I itu.
(C) Nilai rupiah saat ini sangat
menggembirakan.
(D) Yuanita memilih mematikan
1. . Tanaman yang termasuk dunia lampu.
jamur adalah kamir, kapang, dan (E) Mereka menegaskan bahwa acara
cendawan. Jamur menyerap mineral tersebut akan dihentikan.
dan nutrisi dari bahan organik. Jamur
bukan tumbuhan karena tidak
berfotosintesis. Jamur mikoriza 4. Aku bagaikan kejatuhan bulan.
memiliki hubungan simbiosis dengan Kata kejatuhan dalam kalimat di atas
tumbuhan. Jamur hidup di dalam akar berstruktur sama dengan kata
tumbuhan dengan cara menyerap berimbuhan ke-an dalamkalimat
nitrogen dari tanah dan menukarnya berikut, kecuali ....
dengan karbon pada tumbuhan. (A) Pak Dahlan kebakaran jenggot
Jumlah nomina yang terdapat pada mendengar namanya disebut-sebut.
kalimat kedua adalah …. (B) Tetangga saya tidakmengira
(A) 2 kedatangan tamu tak diundang tadi
(B) 3 malam. (C) Mata adikku kemasukan
(C) 5 debu sehingga memerah.
(D) 6 (D) Tiada yang risau akan mematian
(E) 7 koruptor yang menyengsarakan
. negara.
(E) Baru saja ia kepergian suaminya.
2. Asri sedang membersihkan
halaman rumahnya. Makna imbuhan
me-kan pada kalimat di atas sama 5. Pada malam Minggu
maknanya dengan imbuhan me-kan diselenggarakan pertunjukkan musik
pada kalimat... di panggung utama. Kata
(A) Penjaga gawang itu melemparkan pertunjukkan mengalami pola
bola kepada temannya. pembentukan yang sama dengan kata
cakan adik sebuah cerita anak-anak. berimbuhan per–an dalam kalimat
(C) Rudi mengirimkan bingkisan itu berikut, kecuali ....
nya yang berada di luar kota. (A) Peragaan busana kali ini
(D) Ayah sedang meninggikan pagar menampilkan busana pengantin.
halaman. (B) Saya selalu berupaya memberikan
kan buku perpustakaan kepada perhatian kepada orang tua.
temannya karena ia tidak dapat hadir (C) Ani mengagumi perlakuan Ria
di sekolah. kepada sahabatnya yang cacat.
(D) Persamaan hak kaum perempuan
3. Makna men-kan pada kalimat dengan pria terus diperjuangkan.
“Pemusik Jazz, Benny Likumahuwa, (E) Perbuatan Rio terhadap temannya
mempromosikan Ambon sebagai kota sulit dimaafkan
yang aman dan memiliki pesona “
sama dengan makna men-kan pada .... 6. Kata-kata di bawah ini terdiri dari
(A) Penerbit memublikasikan buku 2 morfem, kecuali …
baru Dewi Lestari. (A) Bernyanyi
(B) Kelalaian
(C) Menandatangani resiko terkena penyakit
(D) Kedepankan degeneratif.
(E) Pelarian (B) Untuk mengurangi risiko terkena
penyakit degenertif, kita harus
merubah kebiasaan makan
makanan yang kaya lemak ke
7. Proses pengimbuhan pada kat makanan kaya serat.
berpakaian sama dengan proses (C) Rata-rata penduduk Indonesia
pengimbuhan pada kata… mengonsumsi serat 10,5 gr per
(A) Berkeliaran hari.
(B) Bertujuan
(D) Jumlah rata-rata konsumsi serta
(C) Berlarian
(D) Berdandan penduduk Indonesia baru
(E) Bersamaan sepertiga dari konsumsi serat
yang dianjurkan.
(E) Kebutuhan serat 25-35 gr per
8. Kata-kata berikut dibentuk hari dapat terpenuhi apabila kita
dengan prefiks ber-an, mengonsumsi 2-3 porsi nasi dari
KECUALI….
beras tumbuk yang masih ada
(A) Berpakaian
(B) Bermunculan kulit arinya, 1-2 porsi biji-bijian,
(C) Berenang dan 6 porsi buah dan sayur.
(D) Berhamburan
(E) Berlawanan’ 11. Kalimat yang tidak benar
pembentukan katanya adalah..
(A) Ia memakai baju yang mencolok
9. Fungsi imbuhan ke-an pada warnanya.
kalimat di atas sama dengan (B) .Proses penyucian hama itu
fungsi –nya pada kalimat … memerlukan suhu tinggi.
(C) Banyak orang menyuci pakaian
(A) Dia mengecat rumahnya dengan di Kali Ciliwung.
cat putih. (D) Anak-anak sedang belajar
(B) Hasil kebunnya cukup untuk mengarang.
biaya hidup keluarganya (E) Si Banu menjolok mangga di
(C) Buku itu berjudul lahirnya kebun Pak Ahmad.
Pancasila.
(D) Sakitnya tidak tertahankan
olehku. 12. (1) Saat ini, pertumbuhan
(E) Obat itu bukan main mahalnya. ekonomi Indonesia yang berada
pada kisaran 5 – 6 persen. (2)
Apabila pertumbuhan itu dapat
10. Kalimat yang mengandung
dipertahankan akan menambah
bentukan kata yang salah
jumlah masyarakat kelas menengah
terdapat pada
hingga 90 juta orang. (3)Apabila
(A) Perubahan pola makan
kita mampu mendorong
tradisional menjadi modern yang
pertumbuhan sampai 7 persen,
umumnya rendah serat dan tinggi
jumlah itu akan bertambah lagi
lemak harus dibayar dengan
hingga 170 juta orang.
(4) Apabila potensi ini tidak (C) Kerja
diwujudkan dalam aksi dan (D)Konjungsi
memomentum yang baik dilewatkan (E) Keterangan
begitu saja karena kita begitu asyik
dengan urusan lain, prediksi para
15. Saya bersedia belajar dengan
investor tersebut tidak akan menjadi
baik Jumlah morfem bebas dan
kenyataan. (5) Tentunya pilihan ada
terikat adalah
di tangan kita semuanya. Kalimat
(A) 5 MB dan 2 MT
yang memuat pronomina adalah …
(B) 4 MB dan 2 MT
(A) (1), (2), (3)
(B) (3), (4), (5) (C) 5 MB dan 4 MT
(C) (1), (3), (4) (D) 4 MB dan 3 MT
(D) (2), (4), (5) (E) 4 MB dan 5 MT
(E) (1), (3), (5)

13. Pola pembentukan kata


pemukiman penduduk dalam
kalimat Banjir merupakan satu
ciri banyak kota dan tempat lain
yang padat permukiman
penduduk di hampir sepanjang
pantai utara Pulau Jawa sama
dengan pola pembentukan kata
berikut, KECUALI .
(A) perakitan mobil
(B) perdagangan luar negeri
(C) perdamaian dunia
(D) perlombaan bintang radio
(E) perundingan tingkat menlu

14. Karbon adalah unsur kimia


nonmental yang disimbolkan
dengan huruf C. Karbon berada
di alam dapat dikelompokkan
sebagai karbon murni dan terikat
secara kimia dalam senyawa
alam yang dapat berbentuk
kristal murni.
Kata yang dicetak miring dalam teks
di atas termasuk dalam jenis kata
…..

(A)Sifat
(B) Bilangan
SOAL PENGANTAR 6 4.Pola pembentukan kata
mengetengahkan tidak terdapat pada
MORFOLOGI II kata...
(A) Mengejawantahkan
(B) Mengebumikan
1.Kata berimbuhan me-I yang (C) Mengedepankan
menyatakan ‘perbuatan yang dilakukan (D) Mengesampingkan
secara berkali-kali’ terdapat dalam (E) A,B,C,dan D BENAR
kalimat...
(A)Petugas sensus akan mendatangi
para responden minggu ini. 5. Morfem –nya sebagai akhiran terdapat
(B)Kami belum mengetahui pada kalimat berikut...
rencana keberangkatan mereka ke Paris. (A) Siapa saja yang menemukan
(C)Saya harus segera melengkapi ATM-ku kuberikan hadiah ini
berkas lamaran ini. kepadanya.
(D)Kusir dokar itu mencambuki (B) Pekerjaan rumahnya tertinggal di
kudanya sepanjang jalan rumah.
.(E)Kesebelasan kami menempati (C) Rumah itu dijualnya dengan
posisi juru kunci. hargamurah.
(D) Terhadap pencopet itu, mereka
memukulnya beramai-ramai.
2.Afiks per-dapat menyatakan makna (E) Maraknya tindak kriminalitas
berikut, kecuali... makin menggelisahkan
(A) orang yang di... masyarakat.
(B) orang yang me...
(C) alat untuk me...
(D) menyatakan paling 6. Cipratan air itu mengotori pakaiannya.
(E) mempunyai sifat yang tersebut Afiks pada kata mengotori bermakna
dalam bentuk dasar gramatikal sama dengan...
(A) Melempari
(B) Mengairi
3.Makna gramatikal prefiks ter-adalah (C) Menerbangkan
sebagai berikut, kecuali... (D) Mengandangkan
(A) orang yang di... (E) mengulit
(B) menyatakan tingkat paling
(C) menyatakan dapat di...
(D) perbuatan yang tidak disengaja
(E) menyatakan proses 7. Sufiks –nya yang berfungsi sebagai
kata ganti empunya (kepunyaan)
terdapat pada kalimat...
4.Afiks ke-an pada kata di bawah ini (A) Minumlah obatnya sehari tiga
berfungsi membentuk kata benda, kali!
kecuali.. (B) Akhirnya, datang juga anak itu.
(A) kebaikan (C) Bagaimanapun pentingnya
(B) kebersihan urusan itu, tundalah dulu!
(C) ketuhanan (D) Keesokan harinya ia pun
(D) .kepergian pulanglah.
(E) .kelihatan (E) Peluangnya tinggal sekali lagi.
8.Yang termasuk afiks produktif 12.Pemberian konfiks yang benar
dalam bahasa Indonesia terdapat pada pada kata majemuk adalah seperti
kata... contoh berikut, kecuali...
(A) Ibunda (A) Pertanggungan jawab
(B) Gemetar (B) Keikutsertaan
(C) Seniman (C) penitikberatan
(D) Gerakan (D) keserbagunaan
(E) Rohaniah (E) pengalihanbahasaan

13.Fungsi afiks yang terdapat dalam


9.Paman telah mencarikan kakak- kata terhukum terdapat pula dalam
kakak saya pekerjaan.Pernyataan yang kata berikut,kecuali...
tidak benar sehubungan dengan (A) pembaca
kalimat di atas adalah... (B) masakan
(A) bentuk kakak-kakak (C) permainan
menunjukkan makna intensitas (D) pelempar
kuantitatif (E) terselesaikan
(B) predikat berjenis verba
dwitransitif 14. Kata kerja berkonfiks ke-/-an yang
(C) terdapat aspek/ kala perfektif berkaitan dengan kata kerja pasif
(D) mengandung makna benefakti berafiks di-/-i, terlihat dalam kalimat-
(E) mengandung frasa adjektiva kalimat berikut, kecuali...
(A) Kasihan sekali anak itu, dia
10.Hakim mempertemukan kedua gegar otak karena
orang yang bersengketa itu. Makna kejatuhankelapa.
gramatikal kata mempertemukan (B) Rumah Pak Jumangin semalam
adalah... kedatangantamu yang tidak
(A) membuat jadi bertemu diundang.
(B) menyuruh bertemu (C) Pintu rumahnya kedapatansudah
(C) supaya bertemu terbuka.
(D) membuat pertemuan (D) Puncak Merapi sudah tidak
(E) berusaha mempertemukan kelihatanlagi dari sisi sekarang.
(E) Mobilnya kehabisanbensin dalam
perjalanan pulang.
11.Kalimat yang tidak benar
pembentukan katanya adalah... 15.Ibu membacakan adik
(A) Ia memakai baju yang dongeng.Fungsi imbuhan me-kanpada
mencolok warnanya. kata membacakan sama dengan imbuhan
(B) Proses penyucian hama itu -me-kan dalam kalimat...
memerlukan suhu tinggi. (A) Pak Husni menjahitkan kain
(C) Banyak orang menyuci pakaian sarungnya yang baru.
di Kali Ciliwung. (B) Ayah membelikan adik sepeda.
(D) Anak-anak sedang belajar (C) Pemuda-pemuda mengucapkan
mengarang. sumpahnya dalam upacara tadi
(E) Si Banu menjolok mangga di pagi.
kebun Pak Ahmad. (D) Pelayan menyediakan makan
pagi.
(E) Peristiwa itu sangat
menyedihkan.
SOAL PENGANTAR 7 4. Imbuhan ber- yang arti
mempunyai terdapat pada kalimat...
MORFOLOGI III (A)Adik bersepeda ke sekolah.
1. Makna imbuhan ber- pada kalimat (B)Murid berusaha menyelesaikan
Ia bercukur setiap pagi dapat tugasnya.
dijumpai pada kalimat...
(C)Saya bermaksud mengunjungi ibu.
(A) Setiap hari kerjanya hanya
berhias saja. (D)Ibu berbelanja di pasar.
(B) Gadis cantik itu bernama
Kusumaningtyas. (E)Petinju itu berkeringat selama
(C) Orang yang menolongmu itu bertanding.
bersepatu roda.
(D) Sudah satu bulan ini kami
bersenang-senang saja.
(E) Tidak usah risau, yang penting 5. Penggunaan akhiran –an dalam
hati kita sudah bersalaman. kalimat berikut ini salah, kecuali...
(A)Sudah setahun saya langganan
koran lokal.
2. Makna imbuhan ter- pada kalimat (B)Sekolahan itu dibangun sejak
Bukunya terbawa saya kemarin 1908
adalah... . (C)Di sekolah, Amir kenalan dengan
(A) dalam keadaan di- teman barunya.
(B) tidak sengaja (D)Sayang sekali sepatu
(C) dapat di- yangdibelinya kemarin agak kecilan.
(D) paling (E)Simpulan yang disampaikan
(E) sengaja sudah tepat

3. Afiks yang berfungsi membentuk 6. Kata berlawanan me-yang tidak


adverbia terdapat pada kalimat... menyatakan kerja, terdapat
dalam kalimat..
(A)Bu Marta terkenal sebagai orang (A)Dia tidak mengakui
yang suka berutang, tetapi malas perbuatannya yang salah.
membayar. (B)Penduduk desa itu banyak
yang merotan.
(B)Melihat anjing itu berada di
(C)Para penerjun telah mendarat
belakangnya, dia pun berlari dengan selamat.
secepat-cepatnya. (D)Kami mengontrak rumah di
(C)Peserta SNMPTN tahun depan Pulogadung.
masih berkisar tiga ratusanribu (E)Kami datang menjelang pesta
siswa. dimulai.
(D)Kopi yang dapat
7. Fungsi ber- pada bernafsu dalam
meningkatkan kesuburandiragukan.
kalimat Ia bernafsu hendak
(E)Ahli kimia itu menerima memukul saya sama dengan fungsi
penghargaan. ber-dalam kalimat...
(A)Berpikir itu pelita hati.
(B)Bertanam padi merupakan
mata pencaharian di desa itu.
(C)Indonesia berambisi 11.Makna imbuhan ber- pada kalimat
memenangkan pertandingan itu. “Ia bercukur setiap pagi” dapat
(D)Setiap hari ia bersepedake dijumpai pada kalimat...
sekolah. (A) Setiap hari kerjanya hanya
(E)Bel tanda masuk sudah berbunyi. berhias saja.
(B) Gadis cantik itu bernama
Kusumaningtyas.
8. Bentuk-bentuk kerancuan afiks (C) Orang yang menolongmu itu
terdapat pada kata-kata di bawah ini, bersepatu roda.
kecuali... bulan ini kami bersenang-senang
(A)memperbesarkan saja.
(B)mempelajarkan (E) Tidak usah risau, yang penting
(C)mengeyampingkan hati kita sudah bersalaman
(D)memperbaiki
(E)diperlebarkan
12.Pembangunan rumah mewah itu
9. Makna afiks pada kata tidak cukup enam bulan.
terbawa dalam kalimat Maaf, Imbuhan pe-an yang semakna
bukumu terbawa kemarin sama dengan imbuhan pean pada kata
dengan... pembangunan tersebut adalah...
(A)Namanya tercantum di antara (A) Karya tulis ilmiah harus ditulis
mereka yang lulus. berdasarkan penelitian.
ni, jalannya masih terseok-seok (B) Penangkapan penjahat itu
setelah ditabrak. dilakukan dengan kekerasan.
(C)Ia duduk tersembunyi di (C) Penyaringan air itu terbuat dari
sudut ruangan itu. sabut kelapa dan arang.
(D)Ani teringat akan ibunya. (D) Pendapatan pengamen itu bisa
(E)Ibu tertidur di ruang tamu seratus ribu per hari.
menantikan ayah tak kunjung (E) Pendidikan anak itu hanyasampai
pulang. bangku SMP.

1. Bangsa Indonesia ... nilai


budaya luhur peninggalan nenek 13. Membangun sebuah kota perlu
moyangnya. pendekatan secara manusiawi.
Kata yang tepat untuk mengisi Artinya perlu perencanaan yang baik
bagian rumpang di atas adalah... dengan memahami masa lalu dan
(A)mewariskan peruntukannya di masa depan.
(B)mewarisi Nomina dasar yang terdapat pada
(C)warisan penggalan teks di atas adalah ….
(D)pewaris (A) Pendekatan, perencanaan,
(E)ahli waris peruntukan
(B) Manusiawi, kota, masa
(C) Kota, masa
(D) Sebuah, secara
(E) Membangun, memahami
14.Bom meledak di depan kedutaan
Australia. Kata berimbuhan per-an
yang sama maknanya dengan kata
berimbuhan ke-an pada kalimat di
atas terdapat dalam kalimat... .
(A) Kedua anak yatim itu mendapat
perlakuan yan tidak selayaknya.
(B) Para artis selalu mengenakan
pakaian mengikuti perkembangan
mode.
(C) Perbaikan jalan di desa itu
dibiayai oleh pemerintah.
(D) Permukiman penduduk di
bantaran sungai ituselalu dilanda
banjir.
(E) Orang yang beragama tidak
akan melakukan perbuatan
yang amoral.

15. Kata yang merupakan hasil proses


nominalisasi dari verba adalah ….

(A) Pekerjaan,pemukulan,
penjernihan
(B) Kecantikan,kesedihan,
kegembiraan
(C) Berlima, bersatu, bersepuluh
(D) Merajai, menggurui, meracuni
(E) Mempercantik, mempekerjakan,
memukul
SOAL PENGANTAR 8 (D) Mikroplastik, Serangga,
Makhluk Hidup
KETERAMPILAN BAHASA I (E) Cara Baru Penyebaran
Mikroplastik
Paragraf unuk menjawab pertanyaan
nomor 1 s.d. 3!
2. Kata berimbuhan yang salah
(1)Penelitian terbaru menyatakan
digunakan pada paragraf 2 adalah ….
bahwa larva nyamuk yang menelan
mikroplastik dapat menyebarkan (A) menelan
mikroplastik ke rantai makanan manusia. (B) mengamati
(2)Diketahui mikroplastik, potongan (C) menunjukan
kecil plastik yang terpecah dari produk (D) memasuki
buatan manusia, seperti pakaian sintetis, (E) menemukan
ban mobil, dan lensa kontak, memenuhi
lautan dunia saat ini. (3)Karena sulit 3. Kata yang paling tepat untuk
dideteksi, mikroplastik dapat melengkapi titik-titik pada kalimat 6
menimbulkan masalah serius bagi hewan adalah ….
laut.
(A) tertransfer
(4)Baru-baru ini para peneliti (B) berpindah
dari Universitas Reading menemukan (C) teralihkan
bukti yang menunjukan bagaimana (D) tergantikan
mikroplastik dapat memasuki ekosistem (E) berubah
manusia melalui nyamuk dan serangga
4.Satu pelajaran penting yang dapat
terbang lainnya. (5)Tim peneliti tersebut
diambil dari pelbagai macam upaya
mengamati larva nyamuk yang menelan
penyelesaian konflik Palestina dan
microbeads–jenis mikroplastik yang
Israel adalah bahwa konferensi atau
biasa ditemukan pada produk kosmetik
perundingan apa pun tidak mampu
—sebelum mengamati siklus
mengubah warna jalan-jalan di Timur
kehidupannya. (6)Mereka menemukan
Tengah. Tatkala perdamaian menderu
fakta bahwa banyak sekali partikel
kencang di ruang konferensi, suara
plastik yang [...] ke tubuh nyamuk
tembakan dan kekerasan acap kali
dewasa.
mewarnai jalan-jalan di kawasan itu.
Persoalan konflik Palestina dan Israel
bukan persoalan elite belaka. Persoalan
1. Judul yang paling tepat untuk ini telah menyebar dan meluas menjadi
melengkapi tulisan di atas adalah …. persoalan masyarakat Timur Tengah
secara umum. Berdasarkan isi bacaan di
(A) Serangga Pemakan Mikroplastik
atas, pernyataan berikut ini benar,
(B) Bahaya Mikroplastik bagi
kecuali ....
Tubuh Manusia
(C) Penyebaran Mikroplastik oleh
Serangga
(A) Konflik Palestina dan Israel (A) Penurunan pertumbuhan
bukan hanya persoalan kedua ekonomi di Amerika Serikat
bangsa itu. berdampak pada penurunan
(B) Dalam penyelesaian konflik pertumbuhan ekonomi global.
Palestina dan Israel antara hasil (B) Pada tahun 2008 pertumbuhan
perundingan dengan kenyataan di ekonomi Amerika Serikat
lapangan sering tidak sejalan. diperkirakan lebih rendah
(C) Sampai saat ini belum ada daripada pertumbuhan ekonomi
perundingan yang dapat global.
menyelesai-kan konflik Palestina (C) Pada tahun 2008 Amerika
dan Israel. Serikat diperkirakan akan
(D) Hanya perundingan para elite mengalami penurunan
kedua negara yang dapat pertumbuhan ekonomi sebesar
menyelesaikan konflik Palestina 1%.
dan Israel. (D) Sebagai dampak penurunan
(E) Masyarakat di Timur Tengah pertumbuhan ekonomi Amerika
belum memiliki pandangan yang Serikat, pertumbuhan ekonomi di
sama dalam menyelesaikan Asia tidak akan turun lebih besar
konflik Palestina dan Israel. daripada besarnya angka
penurunan pertumbuhan
5. Pada tahun 2008 pertumbuhan ekonomi Amerika Serikat.
ekonomi Amerika Serikat diperkirakan (E) Jika pertumbuhan ekonomi
melambat menjadi 2% atau sedikit Amerika Serikat turun 2%,
menurun dibandingkan dengan tahun sebagai dampaknya pertumbuhan
sebelumnya. Menuruf IMF, sekitar 1% ekonomi negara Asia akan turun
penurunan pertumbuhan ekonomi di sebesar 1% - 2%.
Amerika Serikat akan menurunkan
pertumbuhan ekonomi di Asia sekitar 6.Kehidupan orang Malind, salah satu
0,5% - 1%. Dampak resesi global yang suku di Papua yang masih
berasal dari resesi di Amerika Serikat mengandalkan perburuan dan penokokan
akan mempengaruhi proyeksi sagu untuk mencari makanan, sangat
perekonomian di negara-negara Asia, dipengaruhi alam. Selama ribuan tahun
terutama Indonesia. Ekonomi global kekayaan hutan ulayat Malind membuat
diperkirakan mengalami penurunan hidup mereka berkelimpahan. Sagu di
pertumbuhan sebesar 0,4 %, dari 5,2 % mana-mana, saham (kanguru kecil) dan
pada tahun 2007 menjadi 4,8 % pada rusa berlarian, burung- burung berkicau
tahun 2008. Negara-negara di Asia di sekitar kampung adat, dan ikan pun
Tenggara diperkirakan mengalami melimpah. Namun, sekitar du puluh
tekanan paling parah akibat tahun yang lalu, sebuah sentaka budaya
melambatnya ekonomi yang terjadi di terjadi ketika lebih dan 48.000 hektare
Amerika. hutan ulayat dibuka menjadi sawah bagi
para transmigran. Dari bacaan di atas
Berdasarkan bacaan di atas, pernyataan dapat disimpulkan bahwa ....
berikut ini yang salah adalah ....
(A) kehidupan suku Malind telah (E) Itulah sebabnya para peternak
berubah dari sistem meramu dan itik di wilayah itu menunjukkan
berburu ke bercocok tanam. kreativi-tasnya.
(B) pembukaan hutan ulayat menjadi
sawah telah mempengaruhi
kehidupan suku Malind. 8. Asam folat selama ini dikenal luas
(C) kesejahteraan suku Malind sebagai suplemen wajib pada masa
semakin meningkat sejak prakehamilan dan kehamilan ..., timbul
dibukanya hutan ulayat menjadi anggapan bahwa zat ini hanya
sawah. dibutuhkan oleh para calon ibu. ..., baik
(D) sejak dibukanya hutan ulayat pria maupun wanita sama-sama
menjadi sawah, suku Malind membutuhkan asupan harian asam
tidak lagi dapat berburu. folat. ..., kebutuhan harian asam folat
(E) suku Malind belajar bercocok pria dewasa sama besarnya dengan
tanam di sawah dari para kebutuhan wanita dewasa. Kata yang
transmigran yang datang ke tepat untuk mengisi tiga bagian kosong
daerah itu. dalam bacaan di atas berturut-turut
7. Terbatasnya lahan dan modal usaha di adalah ....
tengah semakin tingginya tuntutan (A) lalu, oleh karena itu, sedangkan.
kebutuhan hidup acap kali justru (B) dan, walaupun, adapun.
mendorong kreativitas. Tidak pernah (C) sehingga, padahal, bahkan.
puas dengan hasil kerja yang sama (D) meskipun, akan tetapi, jadi.
bertahun-tahun, terobosan-terobosan (E) karena, sebenarnya, dengan
baru pun dilakukan untuk memperoleh demikian.
hasil yang lebih baik. .... Mereka
berhasil mengembangkan budi daya
ternak itik unggul organik yang
9. Jumlah ini meningkat tajam dari tahun
merupakan persilangan dari itik alabio
sebelumnya yang hanya tiga kasus. (2)
Kalimantan Timur dengan itik Mojosari
Menurut Direktur LBH Palembang, Eti
Jawa Timur.
Gustina, pada tahun 2007 terjadi 39
Kalimat yang tepat untuk melengkapi kasus penyerobotan tanah milik warga di
titik- titik dalam bacaan di atas adalah .... Provinsi Sumatera Selatan. (3) Demikian
salah satu kesimpulan refleksi akhir
(A) Para peternak itik di wilayah itu tahun LBH Palembang. (4) Konflik ini
membuat penemuan baru. di antaranya terjadi di Kabupaten Musi
(B) Demikianlah yang terjadi pada Banyuasin dan Ogan Komering Ilir. (5)
para peternak itik di wilayah itu. Lembaga Bantuan Hukum Palembang
(C) Dua jenis itik disilangkan oleh mencatat peningkatan ketegangan akibat
para peternak itik di wilayah itu kasus penyerobotan tanah milik warga
untuk mendapatkan varietas itik oleh sejumlah perusahaan perkebunan
baru. tahun 2007. Untuk menjadi sebuah
(D) Di wilayah itu para peternak itik paragraf yang runtut, kalimat-kalimat di
menunjukkan kreativitasnya. atas harus disusun menjadi ....
(A) 1-2-4-3-5 yang memiliki rekening bersama lebih
(B) 2-4-3-1-5 aman daripada toko yang hanya
(C) 5-1-3-4-2 menyediakan transfer langsung ke
(D) 2-3-4-1-5 nomor rekening pribadi. Apalagi jika
(E) 5-4-3-2-1 ada sistem COD (cash on delivery),
pembeli dapat langsung membayar saat
menerima dan mengecek langsung
Teks di bawah ini untuk menjawab kondisi produk yang dibelinya.
nomor 10-12!
10.Topik utama pada kutipan tersebut
Hal pertama yang paling adalah ... .
mengganggu pikiran seseorang saat
hendak bertransaksi online adalah (A) cara bertransaksi online
mengetahui keaslian toko. Penipuan (B) penipuan dalam transaksi online
yang sering terjadi dapat dihindari jika (C) nasihat bagi para pelaku bisnis
para calon pelanggan memiliki online
kebijakan dalam membeli. Sebelum (D) hal-hal yang perlu diwaspadai
melakukan pembelian, calon pelanggan dalam transaksi online
sebaiknya mengadakan browsing study (E) cara melakukan bisnis online
melalui internet.
11. Fakta yang terdapat pada kutipan di
Biasanya, bila toko yang dituju atas adalah ... .
itu adalah adalah toko abal-abal, toko
tersebut akan memiliki sejumlah (A) Sebuah produk sangat terkenal
komentar negatif. Hal tersebut harus dijual dengan harga 50% di
diwaspadai oleh pelanggan. bawah harga resmi pasaran.
(B) Toko abal-abal memiliki
Kedua, calon pelanggan perlu komentar negatif.
memeriksa tingkat perbandingan harga (C) Toko online yang memiliki
di toko yang menjadi sasaran. Misalnya, rekening bersama lebih aman.
sebuah produk yang sangat terkenal (D) Para pelanggan perlu melihat
dijual dengan harga 50 % di bawah sistem pembayaran toko online
harga resmi di pasaran. Penawaran itu.
seperti itu harus diwaspadai karena tidak (E) Penawaran di bawah harga
mungkin harga produk yang sama 50 % resmi harus diwaspadai.
lebih murah. Iklan yang terdapat di toko
online tersebut dapat dibuat oleh siapa 12. Kesimpulan yang tepat untuk
saja sehingga diharapkan para pelanggan kutipan di atas adalah ... .
tidak tergiur. (A) Sistem pembayaran langsung di
Selanjutnya, calon pelanggan tempat lebih aman daripada
juga perlu melihat sistem pembayaran mentransfer langsung.
yang dilakukan oleh toko online (B) Pelanggan perlu berhati-hati
tersebut. Toko online yang baik akan dalam melakukan transaksi
menawarkan cara pembayaran yang online.
aman bagi pelanggannya. Toko online
(C) Toko online yang baik adalah politik kontemporer, korupsi
toko yang memiliki sejumlah konvensional digolongkan ke dalam tiga
komentar positif. kategori. (9) Pertama, dari segi
(D) Produk yang ditawarkan di toko pemahaman berdasarkan jabatan publik,
online biasanya lebih murah. korupsi dipandang sebagai
(E) Para pelanggan sering tergiur penyalahgunaan jabatan publik demi
jika ada produk yang rentang keuntungan personal. (10) Dari segi
harganya jauh di bawah harga kepentingan publik, korupsi dilihat
resmi. sebagai tindakan yang merugikan
kepentingan publik. (11) Dari segi
pemahaman pasar, korupsi dilihat
Teks untuk menjawab soal nomor 13-14! sebagai berfungsinya pasar bebas dalam
transaksi kewenangan publik dan tidak
(1) Korupsi institusional merupakan adanya aturan yang jelas untuk mengatur
masalah serius yang mengancam negara tindakan korup itu.
demokrasi karena dapat meruntuhkan
integritas, kinerja, dan kepercayaan 13. Kalimat (2) dalam bacaan di atas
publik terhadap institusi demokrasi. (2) bermaksud menyatakan bahwa ... .
Melalui pendistorsian proses demokrasi (A) Korupsi intitusional berdampak
dan kebijakan publik, dibandingkan lebih parah jika proses
dengan jenis korupsi konvensional, demokrasi mengalami distorsi
korupsi institusional memiliki dampak (B) Penyimpangan proses demokrasi
yang lebih merusak bagi kehidupan dan kebijakan publik berdampak
berbangsa dan bernegara. (3) Akan bagi kehidupan berbangsa dan
tetapi, karena patologi demokrasi ini bernegara.
merupakan bentuk korupsi yang legal (C) Korupsi institusional
dan menjadi bagian formal-integral berdampak lebih buruk bagi
proses pemerintahan, kita sering kali kehidupan bangsa dan negara
mengabaikan dampaknya. (4) Kita daripada korupsi konvensional
telanjur menganggap korupsi jika terjadi penyimpangan
institusional sebagai bagian dari bisnis demokrasi.
yang normal dan aturan main dari proses (D) Jika dibandingkan dengan
demokrasi. (5) Padahal, bila kita tidak korupsi konvensional, korupsi
menangani masalah korupsi institusional institusional berdampak lebih
ini dengan serius, diperkirakan dalam buruk akibat penyimpangan
waktu sepuluh tahun Indonesia akan proses demokrasi dan kebijakan
menjadi negara demokrasi yang gagal. publik.
(6) Perbedaan antara korupsi (E) Distorsi dalam proses demokrasi
institusional dan korupsi konvensional dan kebijakan publik berakibat
terletak pada legalitas tindakan korupsi lebih buruk bagi korupsi
itu. (7) Bila korupsi konvensional institusional daripada korupsi
digolongkan dalam jenis korupsi ilegal, konvensional.
korupsi institusional merupakan bentuk
korupsi yang legal. (8) Dalam ilmu
14. Berdasarkan bacaan di atas, pesan (A) Selain berperan dalam sintesis
yang relevan dengan isi bacaan adalah ... DNA, zat gizi mikro, yakni
. vitamin dan mineral, mempunyai
peran penting dalam perbaikan
(A) Kita harus bersikap hati-hati kerusakan DNA.
terhadap tindakan-tindakan yang (B) DNA sangat sensitif terhadap
sesuai dengan aturan dan kekurangan zat gizi mikro, yakni
prosedur karena tindakan seperti vitamin dan mineral.
itu juga berpotensi korupsi. (C) Kerusakan DNA dapat terjadi
(B) Sebaiknya, korupsi institusional jika kita mengalami kekurangan
dihadapi dengan hati-hati karena zat besi, seng, folat, vitamin B12,
berdampak lebih serius daripada dan kolin, yang semuanya
korupsi konvensional. merupakan zat gizi mikro.
(C) Korupsi institusional jauh lebih (D) Zat gizi mikro, yakni vitamin dan
berbahaya daripada korupsi mineral, sangat penting untuk
konvensional karena korupsi menjaga keutuhan DNA.
institusional dapat berlindung di (E) Pemberian zat gizi mikro, yang
balik aturan yang korup. berupa vitamin dan mineral, telah
(D) Seharusnya lembaga terbukti dapat mengurangi
pemberantas korupsi kerusakan DNA.
memublikasikan aturan-aturan
yang berpotensibmelindungi
koruptor institusional. Para sejarawan umumnya memiliki
(E) Lembaga pemberantas korupsi pandangan yang lebih utuh dalam
perlu lebih memahami potensi melihat suatu fenomena atau peristiwa
korupsi di balik berbagai aturan sosial politik. Berbeda dengan para
mengenai lembaganya pengamat umumnya, para sejarawan
selalu mengaitkan setiap masalah dengan
15. Peran zat gizi mikro adalah sebagai peristiwa jauh ke belakang.
kofaktor enzim pada sintesis
(pembentukan) dan perbaikan DNA, Merle Ricklefs, sejarawan Australia,
pencegahan kerusakan DNA akibat misalnya, membuktikan hal itu dalam
oksidasi, serta pemeliharaan proses buku terbarunya, Polarising Javanese
metilasi DNA. Setiap zat gizi mikro Society. Buku ini merupakan karya yang
dibutuhkan dalam jumlah yang cukup sangat baik dalam melihat gejala
untuk mencapai hasil metabolisme kebangkitan Islam di Indonesia. Ricklefs
maksimal. Kekurangan zat gizi mikro membahas masa‐masa pembentukan
akan mengganggu metabolisme normal kesadaran Islam pada paruh pertama
dan pada akhirnya akan menyebabkan abad ke‐19 hingga awal abad ke‐20.
kerusakan DNA.
16. Topik paragraf di atas adalah…
Kalimat yang cocok untuk membuka
paragraf di atas adalah ... . (A) Pandangan sejarawan dalam
melihat suatu gejala.
(B) Perbedaan pengamatan sportivitas, persaudaraan, dan
sejarawan dengan pengamat perdamaian serta tidak sekadar
umum. merupakan sarana komersial.
(C) Cara sejarawan menganalisis (C) Habitus baru sepak bola
masalah. diperlukan dalam dunia yang
(D) Pembahasan buku baru karangan serba komersial dan penuh
Merle Ricklefs. kekerasan sehingga sepak bola
(E) Pembahasan masa awal harus tanpa batas
pembentukan kesadaran Islam di (D) Slogan sepak bola tanpa batas
Indonesia. mengisyaratkan cita‐cita baru
umat manusia untuk menjaga
perdamaian dan membangun
Sepak bola tanpa batas. Ini memang persaudaraan dan menjauhi
hanya sebuah slogan belaka. Akan kekerasan.
tetapi, slogan itu membersitkan cita‐cita (E) Kekerasan yang kerap
mulia bahwa sepak bola lebih dari membayangi persepakbolaan
sekadar suatu permainan. Sepak bola Indonesia menuntut kita
telah menjadi bagian dari keadaban memahami slogan sepak bola
publik kita. tanpa batas sehingga terciptanya
kedamaian dan persaudaraan.
Melalui sepak bola kita belajar menenun
kain habitus baru untuk menyelimuti 18. Orang dengan mudah mengingat
dunia yang sedang dilanda kedinginan Kosta Rika sebagai Negeri Pisang dan
karena kurangnya solidaritas, minusnya Belanda tenar dengan sebutan Negeri
religiositas, dan hampanya sportivitas. Bunga Tulip. Sementara itu, orang
Mudah‐mudahan harapan Paus Yohanes Selandia Baru senang apabila disebut
Paulus II ketika memberkati stadion berasal dari Negeri Kiwi dan Jepang
Roma menjadi harapan kita juga, masyhur dengan Negeri Sakura. Lalu,
“Jauhilah kekerasan dari bola, cegahlah bagaimana dengan Indonesia?Semula
komersialisasi lewat bola, dan bola saya menduga identitas Indonesia
hendaklah menjadi sarana persaudaraan lebihcocok dengan tanaman padi. Paling
dan perdamaian dunia.” Selamat tidak, sejak kanak-kanak kita dengan
menikmati Piala Dunia. mudah menemukan tanaman padi tidak
jauh dari rumah. Murid-murid sekolah
17. .Kalimat yang tepat untuk meringkas dengan mudah melihat padi dan kapas
seluruh isi paragraf di atas adalah… pada lambang negara. Ungkapan
kemakmuranyang diucapkan pejabat
(A) Sepak bola merupakan habitus
juga terkait dengan pertanian tanaman
baru masyarakat dunia sekarang
padi sehingga memunculkan citra di
untuk mengurangi solidaritas,
dalam benak bahwa negeri ini adalah
sportivitas, religiositas manusia
negeri tanaman padi. Akan tetapi,
yang semakin lemah.
identitas ini tidak mewakili citra seluruh
(B) Sepak bola dapat menjadi sarana
negeri ini. Tanaman padihanya
untuk membangun solidaritas,
ditemukan di wilayah tertentu di
Sumatera,Jawa, dan sebagian kecil (D) )Bahasa Indonesia sebagai
Sulawesi bagian selatan.Apa kesimpulan bahasa nasionalkurang
yang dapat ditarik dari wacana diatas? diperhatikan siswa SMA Jakarta.

(A) Setiap negara memiliki identitas (E) Pada umumnya siswa SMA
berdasarkanhasil produksi Jakarta tidakmenggunakan
terkenal negara tersebut. bahasa Indonesia yang baik
(B) Indonesia dikenal sebagai negara danbenar.
padi karenapadi menjadi salah
satu unsur lambang negara. 20. Sejalan dengan arah pelaksanaan
(C) Padi dapat ditemukan di seluruh politik luar negeriRI serta sekaligus
wilayahIndonesia sehingga dalam upaya menyerap masukandari
menjadi ciri khas kita berbagai pemangku kepentingan,
(D) Identitas Indonesia tidak bisa Departemen Luar Negeri dalam tahun
diwakili padikarena tanaman 2010 telah melakukanberbagai
padi tidak menjadi kegiatan, seperti seminar, sosialisasi,
tanamanutama di seluruh wilayah danpertemuan kelompok ahli, baik di
RI. Jakarta maupundi berbagai
(E) Indonesia tidak memiliki daerah.Gagasan pokok kalimat di atas
identitas sepertinegara-negara adalah ....
lain
(A) Arah pelaksanaan politik luar
19. Topik karya ilmiah: Penggunaan negeri RI.
Bahasa Indonesiadi Kalangan Siswa (B) Arah pelaksanaan politik luar
SMA JakartaLatar belakang untuk negeriDepartemen Luar Negeri
sebuah karya ilmiah yangtepat dan upaya menyerapmasukan
sesuai dengan topik tersebut dari pemangku kepentingan.
adalah .... (C) Kegiatan Departemen Luar
Negeri sejalandengan arah
(A) Pelajaran Bahasa Indonesia pelaksanaan politik luar negeri.
kurang diminatioleh siswa SMA (D) Departemen Luar Negeri
Jakarta sehingga mereka melakukan kegiatan.
tidakmenguasainya. (E) Pertemuan kelompok ahli baik
(B) Nilai bahasa Indonesia siswa di Jakartamaupun di berbagai
SMA Jakarta padaUjian Nasional daerah.
2010 pada umumnya
kurangmemuaskan.
(C) Siswa SMA Jakarta dalam
menulis tidakmemperhatikan
Ejaan Yang
Disempurnakan(EYD).
SOAL PENGANTAR 9 dengan faktor karier atau pekerjaan dan
berubahnya rencana perjalanan atau
KETERAMPILAN BAHASA II liburan akibat pandemi virus corona
covid-19.
(diadaptasi dari:www.liputan6.com)

Bagan batang dan teks untuk menjawab 1. Secara umum, aspek kehidupan
soal nomor 1-7 manakah yang paling terdampak
oleh adanya pandemi Covid 19?
(A) Aspek sosial dan ekonomi
(B) Aspek sosial dan rencana wisata
(C) Aspek ekonomi
(D) Aspek sosial
(E) Aspek keagamaan

2. Pernyataan yang tidak sesuai


dengan isi grafik di atas adalah...
(A) Pandemi memberikan dampak
lebih besar pada kebiasaan
belanja wanita dibanding dengan
pria
(B) Pandemi memberikan dampak
Pandemi memberikan banyak dampak yang sangat besar pada
bagi beberapa aspek kehidupan kehidupan sosial wanita di
masyarakat. Tak terkecuali gaya hidup tengah masyarakat
masyarakat di seluruh tanah air. (C) Pandemi memberikan pengaruh
Perubahan yang paling mencolok terlihat yang lebih besar pada aspek karir
dari cara orang menjalani kehidupan bagi pria jika dibandingkan
sosial dan menggerakkan roda dengan wanita
perekonomian. Di mana masyarakat kini (D) Pendemi memberikan dampak
sangat mengandalkan teknologi digital yang lebih besar pada aspek
untuk tetap menjalani kehidupan sosial wisata daripada aspek karir
dan ekonomi di tengah kebijakan (E) Aspek yang terpengaruh pandemi
physical distancing dan PSBB. Namun paling kecil adalah aspek
benarkah bahwa sektor ekonomi menjadi kebiasaan berbelanja
sektor terbesar yang terdampak negatif
virus corona covid-19? 3. Mengapa aspek sosial menjadi
Sebuah survei dilakukan oleh Snapcart aspek yang paling berpengaruh
untuk menilik seberapa besar dampak dalam masa pandemi?
yang dibawa oleh virus corona terhadap (A) Pandemi mengharuskan
gaya hidup orang Indonesia. Dilakukan masyarakat mengubah pola
pada 17-28 Maret 2020, survei yang sosialiasai dengan adanya jaga
melibatkan 2000 laki-laki dan jarak dan PSBB
perempuan berumur 15-50 tahun di 8 (B) Aspek sosial adalah aspek yang
kota besar di Indonesia (Jakarta, bisa digantikan oleh kemajuan
Bandung, Semarang, Surabaya, Medan, teknologi
Palembang, Makassar, dan Manado) (C) Tradisi masyarakat Indonesia
menunjukkan bahwa pandemi virus yang mewajibkan setiap orang
corona berdampak paling besar terhadap berinteraksi fisik secara
kehidupan sosial masyarakat. Disusul langsung
(D) Aspek sosial adalah aspek yang (E) Bagi kalangan wanita dan pria,
terpenting bagi masyarakat aspek sosial adalah aspek yang
bahkan jika dibandingkan paling banyak mengalami
dengan aspek karir atau perubahan selama adanya
perekonomian pandemi
(E) Aspek kebutuhan belanja dan
wisata adalah aspek sekunder 6. Pernyataan yang sesuai dengan
yang masih dapat paragraf terakhir bacaan tersebut
dikesampingkan oleh adalah...
masyarakat (A) Penelitian tersebut dimaksudkan
untuk mengetahui seberapa besar
4. Informasi berikut ini yang sesuai pengaruh Covid 19 terhadap
dengan isi tabel di atas... aspek sosial masyarakat
(A) Wanita lebih merasa terpengaruh (B) Hasil dari penelitian tersebut
dalam aspek sosial dibandingkan menunjukkan bahwa adanya
dengan pria pandemi virus corona
(B) Pengaruh adanya pandemi memberikan pengaruh paling
terhadap minat berbelanja wanita besar terhadap aspek sosial
sama dengan pria masyarakat
(C) Pandemi memberikan pengaruh (C) . Adanya pandemi virus corona
yang sama besar pada aspek tidak memengaruhi rencana
kegamaan pria dan wanita perjalanan wisata beberapa
(D) Pria lebih merasa pandemi masyarakat
memengaruhi rencana liburan (D) Survey untuk data penelitian
daripada wanita tersebut dilakukan di seluruh
(E) Wanita lebih merasa terpengaruh kota di Indonesia
dalam aspek belanja (E) Tujuan utama dilakukannya
dibandingkan dengan pria penelitian adalah melihat
seberapa besar pengaruh
5. Informasi yang tidak tepat sesuai kesehatan pada keberlangsungan
dengan isi grafik di atas adalah... aspek sosial masyarakat

(A) Bagi kalangan pria, pandemi 7. Simpulan yang paling tepat


memberi pengaruh berbelanja untuk bacaan di atas adalah...
yang tidak terlalu besar yaitu (A) Pandemi virus corona
20% jika dibandingkan dengan memberikan pengaruh yang
wanita besar pada seluruh aspek
(B) Pandemi memberikan pengaruh kehidupan masyarakat Indonesia
yang hampir sama pada aspek (B) Adanya PSBB dan penerapan
karir dan rencana bagi kalangan jaga jarak sebagai upaya
wanita mengatasi penyebaran virus
(C) Aspek keagamaan sangat korona mengakibatkan aspek
terpengaruh oleh adanya sosial menjadi aspek yang paling
pandemi, baik bagi kalangan terpengaruh di masyarakat
wanita dan kalangan pria (C) Dari seluruh aspek yang
(D) Aspek yang paling sedikit terpengaruh pandemi virus
terpengaruh oleh adanya korona, aspek keagamaan
pandemi dibandingkan dengan merupakan aspek yang paling
aspek lainnya adalah aspek sedikit mengalami pengaruh
kebiasaan berbelanja.
(D) Gaya hidup masyarakat diuji dalam berbagai penelitian," ujar
Indoneisa merupakan bukan satu Brucks. Namun, dia menjelaskan tidak
satunya aspek yang terpengaruh semua burung yang jinak, seperti beo
penyebaran virus corona bisa menunjukkan perilaku prososial.
(E) Indonesia perlu melakukan Pada penelitian sebelumnya,
upaya lebih ketat lagi dalam menunjukkan burung gagak tidak
penerapan PSBB dan jaga jarak membantu rekan-rekannya
agar kehdupan sosial tidak lagi menyelesaikan tugas. Sementara
terganggu penelitian baru terhadap Burung Macaw
Kepala Biru, menunjukkan sikap yang
Teks untuk menjawab nomor 8 – 14! agak egois.
Para peneliti mengatakan temuan
Burung Beo Abu-abu asal Afrika mereka menunjukkan perilaku
ternyata memiliki perilaku unik yang membantu muncul beberapa kali
sebelumnya hanya tampak pada selama evolusi. "Tampaknya tekanan
kelompok primata. Dalam suatu tes yang sosial dan ekologis memiliki potensi
dilakukan oleh para peneliti, seekor sama terhadap mamalia dan juga
Burung Beo Abu-abu Afrika burung, sehingga mengarah pada
ditempatkan berbeda kandang dengan perkembangan yang serupa," jelas
burung yang lain. Melansir The Brucks. Makalah yang ditulis Brucks
Guardian, dalam tes tersebut Burung dan timnya dalam jurnal Current
Beo Afrika diberi makan dan Biology juga melaporkan bagaimana
menunjukkan perilaku prososial. Dalam pengujian dilakukan pada dua
tes tersebut, burung tersebut justru spesies burung nuri. Kedua spesies
membagikan makanan yang dimiliki burung tersebut diberi tugas untuk
kepada rekan-rekannya. Perilaku yang melewati token dalam bentuk cincin
ditunjukkan Burung Beo Afrika tersebut, logam. Melalui lubang di
sebelumnya hanya tampak pada kera. kompartemen mereka ke burung
Burung tersebut membantu rekan- tetangga dari spesies yang sama.
rekannya untuk menyelesaikan tugas, Token tersebut kemudian diteruskan
meskipun tugas itu tidak oleh burung kedua ke manusia,
menguntungkan untuk diri mereka melalui lubang lain, yang akan
sendiri. Dalam studi pertama ditukar dengan sepotong kenari.
menunjukkan burung memperlihatkan Total ada delapan Burung Beo
perilaku membantu tanpa pamrih. Afrika dan enam Macaw Kepala
Sementara perilaku prososial lainnya Biru yang terlibat dalam percobaan
terlihat pada burung ini. Tim peneliti itu. Semua dilatih secara individu
mengatakan perilaku membantu rekan- untuk bertukar token dengan
rekannya untuk mencapai suatu tujuan makanan ketika seorang manusia
disebut bantuan instrumental. mengulurkan tangannya. Para
Sebelumnya, perilaku tersebut hanya peneliti melihat Burung Beo Afrika
ditunjukkan pada orangutan dan membantu burung yang lain dengan
bonobos. Dr. Désirée Brucks, rekan memberikan token, ketika manusia
penulis makalah dari Max Planck mengulurkan tangan. Sedangkan
Institute for Ornithology mengatakan pada burung kedua, menjatuhkan
pengujian terhadap Burung Beo Afrika token melalui lubang dan menerima
dinilai sangat tepat. "Burung beo, camilan untuk dinikmati sendiri.
corvids dan gagak dikenal sebagai Perilaku yang sama juga terlihat,
burung paling cerdas. Mereka disebut ketika peran burung dibalik.
kera berbulu dan mereka telah banyak Semakin banyak token yang
diberikan, maka semakin banyak (C) Mendeskripsikan perilaku
imbalan yang diterima. membantu tanpa pamrih yang
dilakukan kedua hewan tersebut.
8.Pernyataan yang tidak sesuai (D) Menggambarkan keunikan
dengan isi paragraf 1 adalah …… perilaku saling berbagi makanan
(A) Sikap prososial dan saling yang ditunjukkan kedua hewan
membantu ditunjukkan tersebut.
burung Beo Afrika dalam (E) Menunjukkan kesamaan
penelitian. perilaku prososial burung Beo
(B) Burung Beo Afrika Afrika dengan kedua hewan
membantu tanpa pamrih tersebut.
meskipun kadang tidak
menguntungkan.
(C) Orang utan memiliki perilaku 11. Alasan burung Beo dijuluki dengan
yang sama persis dengan kera berbulu adalah ……
perilaku dari burung Beo (A) Burung Beo memiliki perilaku
Afrika. sosial yang sama dengan kera
(D) Burung Beo Afrika memiliki yakni saling membantu.
keunikan perilaku yang (B) Burung Beo memiliki otak
biasanya hanya tampak pada cerdas seperti kera dan banyak
kera. diuji dalam berbagai penelitian.
(E) Tidak hanya berbagi (C) Burung Beo memiliki keunikan
makanan, burung Beo Afrika perilaku tanpa pamrih dalam
juga membantu menyelesaikan pekerjaan.
menyelesaikan tugas. (D) Burung Beo memiliki kebiasaan
saling berbagi makanan yang
9.Berikut ini pernyataan yang benar serupa dengan perilaku kera.
mengenai bantuan instrumental (E) Burung Beo memiliki struktur
adalah …… anatomi seperti kera dan banyak
(A) Perilaku membantu rekan untuk kemiripan dalam perilaku.
menyelesaikan tugas tertentu
(B) Perilaku memotivasi rekan untuk 12. Pernyataan yang benar mengenai
mencapai tujuan tertentu burung gagak sesuai dengan bacaan
(C) Perilaku meminta bantuan dari adalah ……
rekan untuk tujuan tertentu (A) Memiliki otak lebih cerdas dari
(D) Perilaku saling berbagi Corvids.
makanan dengan rekannya (B) Jarang digunakan sebagai objek
(E) Perilaku membantu rekan penelitian
dengan tanpa pamrih (C) Spesies burung yang jinak dan
prososial
10. Tujuan dari penyebutan orang (D) Tidak membantu rekan
utan dan bonobos pada bacaan menyelesaikan tugas.
adalah …… (E) Memiliki sikap yang egois dan
(A) Membandingkan interaksi antar anti sosial.
individu pada burung Beo Afrika
dengan kedua hewan tersebut. 13. Salah satu hasil penelitian yang
(B) Menjelaskan perilaku unik saling dilaporkan pada makalah Brucks dan
membantu hanya ditunjukkan timnya adalah ……
oleh kedua hewan tersebut. (A) Dua spesies burung nuri yang
berbeda dipilih sebagai subjek
penelitian tentang perilaku saling Tabel dan teks untuk menjawab soal
membantu. nomor 15 – 20 !
(B) Kedua spesies burung nuri
berhasil menyelesaikan tugas
untuk melewati token dalam
bentuk cincin logam.
(C) Burung nuri pandai menukar
token dengan makanan ketika
seorang manusia mengulurkan
tangannya.
(D) Burung Macaw Kepala Biru agak
egois dengan menerima camilan
buah kenari untuk dinikmati
sendiri.
(E) Burung nuri berperilaku
membantu burung lain dengan
memberikan token saat manusia
mengulurkan tangan

14. Pernyataan yang tidak sesuai


dengan bacaan yang diberikan
adalah ……
(A) Total terdapat 14 ekor burung
nuri dari dua spesies berbeda
yang terlibat dalam percobaan
bertukar token dengan makanan. Dikutip CNN Indonesia, Direktur
(B) Tekanan sosial dan ekologis Statistik Kependudukan dan
disinyalir memiliki potensi Ketenagakerjaan BPS Razali Ritonga
pengaruh yang sama terhadap mengatakan jumlah angkatan tenaga
perilaku mamalia dan juga kerja meningkat sedangkan daya serap
burung.
tenaga kerja dari beberapa industri
(C) Saat peran burung dibalik terjadi
perubahan perilaku yang melemah. Jumlah angkatan kerja pada
ditunjukkan oleh burung Beo Agustus 2015 bertambah 510 ribu orang
Afrika dan burung Macaw menjadi 122,38 juta, dibandingkan
Kepala Biru. Agustus 2014 yang sebanyak 121,87 juta
(D) Perilaku saling membantu adalah jiwa. "Ada PHK dan daya serap yang
hal yang unik bagi spesies agak menurun, sehingga pengangguran
burung dan biasanya hanya
meningkat," kata Rizali di kantor pusat
ditunjukkan oleh primata seperti
kera. BPS, Jakarta, Kamis (5/11). Razali
(E) Meskipun tidak jinak spesies mengatakan sebagian industri yang
burung gagak banyak diuji dan melakukan PHK adalah industri yang
menjadi subjek dalam berbagai memiliki ketergantungan terhadap bahan
penelitian karena memiliki otak baku impor. Melemahnya nilai tukar
cerdas. rupiah terhadap dolar AS turut
menambah beban biaya produksi sektor
industri tersebut.
15. Pernyataan yang sesuai dengan nilai tukar rupiah terhadap nilai
isi paragraf 1 adalah …… dolar AS.

(A) Terjadi peningkatan angka 17. Pernyataan yang benar mengenai


pengangguran sekitar 10% tabel jumlah pengangguran di
dibandingkan tahun sebelumnya. Indonesia adalah……
(B) Lulusan Sekolah Menengah Atas
memberi kontribusi angka (A) Jumlah angkatan kerja selalu
pengangguran terbesar. mengalami kenaikan setiap tahun
(C) Hanya sebagian kecil lulusan sejak tahun 2013.
Diploma dan Sarjana yang belum (B) Tingkat partisipasi angkatan
memperoleh pekerjaan. kerja yang diukur tiap Agustus
(D) Badan Pusat Statistik merilis memiliki tren naik.
jumlah pengangguran di (C) Banyak angkatan kerja yang
Indonesia setiap bulan Agustus. menganggur tertinggi tercata
(E) Angka pengangguran bertambah pada bulan Februari 2015.
akibat berkurangnya daya serap (D) Tingkat pengangguran terbuka
sektor perindustrian. sangat dinamis, terkadang naik
dan turun.
16. Informasi yang benar tentang isi (E) Dalam periode setahun sejak
dari paragraf 2, kecuali …… Februari 2014, banyak
penganggur naik 300 ribu jiwa.
(A) Peningkatan jumlah angkatan
tenaga kerja tidak dibarengi
dengan peningkatan daya serap 17. Persentase kenaikan jumlah
tenaga kerja dari beberapa pengangguran dari Agustus 2013
industri. hingga Agustus 2015 adalah ……
(B) Dalam kurun waktu satu tahun
(A) 1.24%
terjadi peningkatan jumlah
(B) 2.02%
angkatan kerja sebesar 510 ribu
(C) 3.16%
orang dari semula 121,87 juta
(D) 4.48%
jiwa.
(E) 5.32%
(C) Peningkatan angka
pengangguran akibat daya serap 18. Pekerja tidak penuh dapat
yang menurun mendorong dibedakan menjadi setengah
sebagian industri melakukan penganggur dan pekerja paruh
PHK tenaga kerja. waktu. Persentase pekerja paruh
(D) Pengangguran bertambah akibat waktu berdasarkan data bulan
PHK yang dilakukan oleh Februari 2015 adalah ……
beberapa industri dengan
ketergantungan terhadap bahan (A) 71.86%
baku impor. (B) 78.42%
(E) Penambahan beban biaya (C) 80.64%
produksi sektor industri (D) 85.28%
dipengaruhi oleh melemahnya (E) 89.06%
19. Dengan memperhatikan tren
(pola) dari data penganggur dan
pekerja tidak penuh, nilai
perbandingan yang mungkin antara
penganggur dan pekerja tidak penuh
pada Februari tahun 2016 adalah
……

(A) 1 : 4
(B) 2 : 11
(C) 3 : 13
(D) 2 : 9
(E) 1 : 5

20 Berikut ini pernyataan yang tidak


sesuai dengan tabel adalah ……

(A) Banyak angkatan kerja yang


bekerja dan tercatat di bulan
Februari selalu lebih tinggi dari
catatan di bulan Agustus.Tingkat
(B) partisipasi angkatan kerja
berkisar antara 66% berdasarkan
catatan di bulan Agustus setiap
tahunnya.
(C) Tingkat pengangguran terbuka
dihitung dari nilai persentase
penganggur terhadap angkatan
kerja yang bekerja.
(D) Sebagian kecil pekerja tidak
penuh merupakan setengah
penganggur dengan persentase
sekitar 30 persen.
(E) Meskipun angkatan kerja terus
mengalami kenaikan tiap tahun,
pekerja tidak penuh justri
mengalami penurunan.
SOAL PENGANTAR 10 Penggunaan bahan bakar fosil pada
kendaraan bermotor merupakan salah
KETERAMPILAN BAHASA III satu sumber terbesar emisi gas rumah
kaca. Penggunaan kendaraan bermotor
Bacalah teks berikut untuk mengerjakan yang tidak efisien seperti penggunaan di
soal nomor 1-5! bawah kapasitas angkut atau
penggunaan untuk jarak pendek, akan
(P1) Isu perubahan iklim
meningkatkan emisi karbondioksida
merupakan isu global sehingga dalam
secara signifikan. Perilaku masyarakat
penanganannya perlu melibatkan seluruh
yang konsumtif dikombinasikan dengan
pihak secara global. Upaya pengelolaan
industri otomotif tanpa batasan produksi
lingkungan saat ini belum berjalan
membuat masyarakat selalu ingin
maksimal karena banyak kendala yang
memiliki produk terbaru keluaran
dihadapi. Pemasalahan yang muncul
perusahaan otomotif. Tingginya
adalah jika ada gerakan mencegah global
permintaan pasar mendorong
warming berskala besar berarti akan
meningkatnya produktivitas sehingga
terjadi perubahan pada struktur ekonomi,
perusahaan akan berinovasi membuat
sosial, dan budaya. Hal tersebut menjadi
produk yang menarik perhatian
kendala lantaran merupakan sektor yang
konsumen.
berkaitan langsung dengan lingkungan
hidup manusia. Fenomena iklim global (P3) Menghadapi isu lingkungan
terjadi sebagai bagian dari hasil kegiatan global, pemerintah mengeluarkan
manusia, khususnya dalam sektor kebijakan mobil murah ramah
ekonomi. Kemampuan aparat lingkungan. Namun kebijakan ini
pemerintah dalam pengelolaan disinyalir berdampak negatif pada sektor
lingkungan perlu ditingkatkan dengan transportasi darat dan energi. Penciptaan
memberikan informasi lebih intensif produk berlabel ramah lingkungan tanpa
mengenai isu lingkungan seperti disertai aturan pembelian dan
perlindungan atmosfer dan perubahan pembatasan produksi kendaraan akan
iklim dengan tujuan mengurangi dampak membuat jalan-jalan besar dipenuhi
negatif yang ditimbulkan. Apabila ingin kendaraan dari berbagai jenis. Inovasi
menghentikan laju degradasi yang berkonsepkan ramah lingkungan
lingkungan, perubahan juga harus akan mengurangi ketergantungan
dilakukan dalam pola kehidupan penggunaan bahan bakar fosil karena
manusia. Aktivitas manusia yang tidak digantikan oleh tenaga listrik. Namun
ramah dengan lingkungan akan transformasi bahan bakar tersebut akan
mengakibatkan kerusakan alam sehingga meningkatkan penggunaan energi listrik.
dampaknya akan kembali kepada Padahal pembangkit tenaga listrik
manusia sendiri. Dengan adanya menjadi penyumbang emisi terbesar bagi
teknologi ramah lingkungan, manusia lingkungan. Jika mobil ramah
dapat turut serta menyelamatkan lingkungan menggunakan tenaga listrik
lingkungan sekaligus memenuhi maka justru melipatgandakan
kebutuhan mereka yang memiliki penggunaan tenaga listrik, dan tentunya
ketergantungan besar terhadap teknologi juga akan melipatgandakan emisi yang
dihasilkan pembangkit tersebut. Emisi
. (P2) Sumber utama polusi udara
karbon akan meningkat dengan pesat
di kota-kota besar adalah kendaraan
jika batu bara digunakan untuk
bermotor. Semakin macet kondisi lalu
menghasilkan tenaga listrik, bahkan
lintas, semakin besar polusi udara yang
pembuatan mobil bertenaga listrik lebih
ditimbulkan yang pada akhirnya
menghasilkan limbah yang cukup
memperburuk kualitas udara.
berbahaya dibandingkan pembuatan (D) Pencegahan global warming
mobil konvensional. berskala besar belum bisa
Sumber: https://www.kompasiana.com/ dilakukan karena pengelolaan
lingkungan belum dapat
maksimal.
(E) Belum ada gerakan pencegahan
1. Berdasarkan paragraf 1, simpulan global warming berskala besar
yang benar adalah …… dan upaya pengelolaan
(A) Suatu masalah bukan isu global lingkungan belum dapat
apabila tidak diperlukan maksimal.
penanganan seluruh pihak global. 3. Berdasarkan paragraf 1, apabila
(B) Suatu masalah adalah isu global tidak dilakukan perubahan pada
apabila masalah tersebut terkait pola kehidupan manusia dan
dengan perubahan iklim manusia tetap berperilaku tak
(C) Suatu masalah adalah isu global ramah dengan lingkungan,
apabila diperlukan penanganan simpulan yang paling mungkin
seluruh pihak secara global. benar adalah …
(D) Penanganan seluruh pihak global (A) Kerusakan alam akan terjadi
tidak diperlukan apabila suatu sehingga manusia akan
masalah bukan isu global. menerima dampak dari
(E) Suatu masalah bukan isu global perilakunya sendiri.
apabila masalah tidak terkait (B) Degradasi lingkungan
dengan isu perubahan iklim. berdampak buruk pada manusia
dan kerusakan alam semakin
2. Berdasarkan paragraf 1, apabila banyak terjadi.
terdapat banyak kendala dalam (C) Kerusakan alam sebagai bentuk
pengelolaan lingkungan meskipun degradasi lingkungan akan
tidak terjadi perubahan pada berdampak negatif pada manusia.
struktur kehidupan manusia, (D) Degradasi lingkungan akan tetap
simpulan yang paling mungkin terjadi dan manusia akan
benar adalah …… menerima dampak dari
(A) Pengelolaan lingkungan belum kerusakan alam.
maksimal dan banyak kendala (E) Kerusakan alam terjadi dan
dalam gerakan pencegahan manusia mendapatkan dampak
global warming berskala besar. negatif dari degradasi lingkungan
(B) Pengelolaan lingkungan belum
berjalan maksimal atau belum
ada gerakan pencegahan global
warming berskala besar. 4. Berdasarkan paragraf 2, apabila
(C) Kurang maksimalnya kondisi lalu lintas padat oleh
pengelolaan lingkungan karena kendaraan dan mendorong
pencegahan global warming penggunaan bahan bakar fosil
berskala besar masih belum yang lebih besar, simpulan paling
dilakukan. mungkin benar adalah……
(A) Lalu lintas macet dan kualitas Bacalah teks berikut untuk mengerjakan
udara buruk akibat dari emisi gas soal nomor 6 - 8 !
rumah kaca.
(B) Tingkat polusi udara meningkat
dan semakin memperburuk (P1) SMK merupakan
kualitas udara. pendidikan menengah vokasi yang
(C) Emisi gas rumah kaca semakin dianggap sebagai pencipta sumber daya
meningkat dan kualitas udara manusia terampil dan bermutu, masih
semakin buruk. harus menghadapi rentetan panjang
(D) Kualitas udara memburuk dan persoalan. Idealnya, sekolah menengah
bertambahnya sumber emisi gas kejuruan (SMK) didesain untuk
rumah kaca. menciptakan lulusan yang siap masuk
(E) Lalu lintas macet dan emisi gas dunia kerja dengan kemampuan teknis
rumah kaca semakin besar akibat yang mereka miliki. Sementara, SMK
kemacetan. justru tercatat sebagai salah satu
penyumbang pengangguran tertinggi.
5. Berdasarkan paragraf 3, simpulan
yang paling mungkin benar adalah (P2) Berdasarkan data Badan
…… Pusat Statistik (BPS) tahun 2019, tingkat
pengangguran terbuka (TPT) selama
(A) Emisi pada lingkungan tidak lima tahun terakhir mengalami
besar apabila pembangkit tenaga penurunan 0,90 persen. Namun, jika
listrik tidak menjadi penyumbang dilihat dari pendidikan yang ditamatkan,
emisi terbesar. tenaga kerja yang tidak terserap pasar
(B) Penggunaan listrik tidak akan kerja paling tinggi justru dari lulusan
berlipat ganda apabila mobil SMK. Pada Agustus 2019, TPT lulusan
ramah lingkungan tidak SMK sebesar 10,42 persen turun 0,82
menggunakan tenaga listrik. persen dari tahun sebelumnya (11,24
(C) Emisi dari pembangkit tidak persen). Persentase itu cukup berjarak
akan berlipat ganda apabila dengan TPT SMA (7,92 persen).
penggunaan tenaga listrik tidak Sementara secara kuantitas, jumlah
mengalami kenaikan. lulusan siswa SMK tahun ajaran
(D) Batu bara digunakan dalam 2018/2019 sekitar 1,4 juta, berada di
pembangkit listrik apabila emisi bawah lulusan SMA yang sekitar 1,5
karbondioksida mengalami juta.
peningkatan yang pesat.
(E) Emisi dari pembangkit listrik (P3) Pengangguran lulusan SMK
semakin meningkat apabila ini menjadi pekerjaan rumah dunia
terjadi transformasi bahan bakar pendidikan, mengingat saat ini ekonomi
menjadi energi listrik Indonesia ditopang oleh sektor industri
dengan kontribusi terhadap
perekonomian nasional hampir 20
persen. Semestinya, lulusan SMK
banyak diserap di sektor industri
manufaktur ini. Sementara itu, (D) 2014-2018
pemerintah kini kian gencar menambah (E) 2015-2019
jumlah SMK untuk menjawab
kebutuhan era industri keempat.
Beberapa strategi digunakan pemerintah 8.Berdasarkan paragraf 2, berapa
untuk memenuhi tantangan itu. persen penurunan lulusan SMK
Pemerintah membekukan penambahan yang tidak terserap pada kerja mulai
SMA, membuka SMK baru, dan bahkan dari tahun sebelum 2019 sampai
mengonversi SMA menjadi SMK. tahun 2019?
Strategi tersebut membuahkan hasil
jumlah SMK dan siswa SMK semakin (A) 0,90 persen
meningkat. (B) 10,42 persen
(C) 0,82 persen
(D) 7,92 persen
6.Berdasarkan paragraf 1 (P1), (E) 11, 24 persen
apabila lulusan SMK tidak siap
untuk masuk dunia kerja dengan
kemampuan teknis yang mereka 9. Pada musim terjadi gelombang
miliki, manakah di bawah ini tinggi di beberapa perairan
simpulan yang PALING MUNGKIN Indonesia. Gelombang tinggi
benar? menyebabkan para pelayanan
tidak dapat melaut. Ketika
(A) Lulusan SMK tersebut tidak nelayan tidak pergi melaut. Ikan
terampil dan bermutu. menjadi langka dipasaran.
(B) Pihak sekolah gagal dalam Kelangkaan ikan menyebabkan
mendidik siswa sehingga tidak harga ikan menjadi sangat mahal.
sesuai dengan tujuannya. Meskipun banyak orang yang
(C) Kemampuan teknis yang ingin membelinya, sebagian,
dimiliki siswa tersebut tidak tidak bisa mengonsumsi ikan
diterima dalam dunia kerja. karaena harganya yang sangat
(D) Lulusan SMK tersebut mahal. Berdasarkan informasi
termasuk dalam penyumbang tersebut, manakah pernyataan
pengangguran di Indonesia. berikut yang PASTI BENAR?
(E) Orang tersebut menjadi
permasalahan yang dimiliki (A) Kelangkaan ikan di pasaran
oleh lulusan SMK pada menyebabkan nelayan tidak
sekarang ini. dapat melaut.
(B) Gelombang tinggi di beberapa
7. Berdasarkan paragraf 2, pada tahun perairan indonesia menyebabkan
berapa tingkat pengangguran harga ikan menjadi sangat mahal.
mengalami penurunan 0,90 persen? (C) Banyaknya ikan dipasaran
menyebabkan banyak orang
(A) .2014 mengonsumsi ikan
(B) 2015
(C) 2019
(D) Banyakya orang yang tidak manakah pernyataan berikut yang
membli ikan menyebabkan PASTI SALAH?
nelayan tidak melaut
(E) Banyaknya orang yang ingin (A) Makanan yang berwarna cerah
mengonsumsi ikan menyebabkan dan menarik tidak memiliki
kelangkaan ikan dipasaran nutris yang diperlukan oleh
tubuh.
10. Sekelompok siswa terdiri atas (B) Orang yang mengonsumsi
beberapa siswa dengan bulan lahir yang makana dengan pewarna
sama. Manakah peristiwwa berikut yang rosela secara berlebihan tidak
memiliki peluang yang PALING mendapat asupan mineral.
BENAR. (C) Orang yang memiliki alergi
sering mengonsumsi makanan
(A) Enam orang dari dua puluh lima dengan pewarna karmoisin
siswa memiliki bulan lahir yang (D) Kanker hati tidak dialami
sama oleh orang yang mengonsumsi
(B) Enam orang dari lima beas siswa makana dengan pewarna
memiliki bulan lahir yang sama rosela
(C) Lima orang dari dua puluh lima (E) Orang yang sering
siswa memiliki bulan lahir yang mengonsumsi makanan
sama dengan pewarna karmoisin
(D) Lima orang dari lima belas siwa tidak mendapatkan asupan
memiliki bulan lahir yang sama protein
(E) Tiga orang dari dua puluh lima
siswa memiliki bulan lahir yang 12. Penderita penyakit tukak lambung
sama mengalami peningkatan gas dalam perut
karena adanya peningkatan kadar asam
11.Orang menggunakan pewarna lambung. Penurunan kadar asam
tambahan agar makanan lebih lambung dapat dilakukan dengan terapi
menarik dan bervariasi. Bunga rosela obat antasida. Menurunnya kadar asam
dan karmoisin adalah pewarna merah lambung dapat mengurangi kadar gas
yang sering digunakan. Meskipun dalam perut karena pH di dalam
warnya tidak terlalu cerah. Bunga lambung menjadi netral. Berdasarkan
rosela mengandung protein, vitamin, informasi tersebut, manakah pernyataan
mineral, antikanker dan peluru batu berikut yang PASTI BENAR?
ginjal. Sementara itu, karmoisin tidak
mengandung nutrisi yang diperlukan (A) Orang yang menderita tukak
tubuh, tetapi memberikan warna yang lambung disembuhkan dengan
lebih cerah, stabil dan tahan lama. meningkatkan asupan antasida
Penggunaan karmoisin yang (B) Orang yang mengalami penyakti
berlebihan dapat menyebabkan alergi tukak lambung memiliki kadar
dan kanker hati. Orang lebih PH tubuh yang lebih rendah dari
disararankan menggunakan bunga orang pada umumnya.
roselesa daripada karmoisin. (C) Penderita penyakit tukak
Berdasarkan informasi terebut, lambung yang melakukan terapi
obat antasida akan mengalami (B) Beberapa warga Kabupaten X
penurunan kadar gas dalam perut mencari pekerjaan ke kota lain
(D) Terapi obat antasida merupakan (C) Warga yang mengganggur
cara paling cepat untuk memutuskan untuk membuka
menurunkan kadar gas dalam lahan bercocok tanam
perut (D) Perusahaan lain juga membuka
(E) Seorang yang mengalami pabrik baru di Kabupaten X
peningkatan gas (E) Pabrik tekstil yang baru dibuka
dalam perut tidak berada di dekat Kabupaten X
mengonsumsi obat antasida

13. Angkutan bus memiliki jadwal yang


lebih tepat daripada angkutan umum Bacalah teks berikut untuk mengerjakan
soal nomor 15 - 20 !
lainnya. Makin tinggi tingkat ketepatan
waktu suatu angkutan umum, makin
besar minat masyarkaat untuk Kuala Lumpur punya ikon
menggunakannya. Berdasarkan menara kembar Petronas, Jakarta punya
ikon Tugu Monas maka Surabaya dan
pernyataan tersebut, manakah yang
Madura punya ikon baru, Jembatan
PALING MUNGKIN menjadi akibat Suramadu menjadi yang terpanjang di
dari tingkat ketepatan waktu angkutan Indonesia. Jembatan ini menghubungkan
bus? pulau jawa (di Surabaya) dan pulau
madura (di Bangkalan). Jembatan
(A) Angkutan bus memiliki tingkatan suramadu dimulai pembangunannya
kenyamanan yang rendah pada tanggal 20 Agustus 2003. Dari total
(B) Angkutan bus dikelola secara panjang jembatan sejauh 5.438 m, terdiri
profesional dari causeway sisi surabaya 1.458 m,
(C) Angkutan bus memiliki resiko causeway sisi madura 1.818 m. Bentang
panjang tengah keseluruhan mencapai
kecelakaan yang tinggi
2.162 m terdiri dari dua Approach
(D) Angkutan bus dipilih oleh Bridge masing-masing 672 m dan Main
banyak orang Bridge sepanjang 818 m. Panjang jalan
(E) Angkutan bus menetapkan harga pendekat di sisi surabaya mencapai 4.35
tiket yang mahal km dan di sisi madura 11.5 km.
Selain konsorsium nasional,
14.Angka pengganguran di Kabupaten X maka yang juga memiliki peranan
menurut dibandingkan tahun penting dalam pembangunan jembatan
sebelumnya. Menurut Ahli ekonomi, suramadu adalah perusahaan konstruksi
pembukaan pabrik tekstil pada tahun ini dari china. Itulah sebabnya alas jembatan
yang disebut dengan Steel Box Grider
berkontribusi pada penurunan tingkat (SBG) pembuatannya dilakukan di
pengganguran di daerah tersebut. tiongkok. Ternyata selain pembuatannya
Manakah pernyataan berkut yang akan harus dilakukan di tiongkok,
MEMPERKUAT argumen ahli ekonomi pengirimannya memakan waktu cukup
tersebut? lama. Pengiriman dari tiongkok
dilakukan empat kali selama 30 hari.
(A) Pabrik tekstil menerima banyak Itupun masih dirakit di Gresik. Berbagai
tenaga kerja dari Kabupaten X tantangan telah dihadapi selama
pelaksanaan konstruksi, mulai dari tahap (D) Jejak pendapat tentang
perancangan, pengeboran sampai pembangunan jembatan
pemasangan grider dan bangunan atas. (E) Hukum yang berlaku di suatu
Kondisi geologis serta kondisi geografis negara
lingkungan proyek yang berada di atas
laut menjadi tantangan untuk dapat 18. Pernyataan berikut yang tidak
menyelesaikan pekerjaan ini. Belum lagi sesuai dengan bacaan diatas adalah
cuaca yang menjadi kendala. Perkiaraan ………
biaya pembangunan jembatan ini (A) Perakitan SBG jembatan
mencapai 4.5 Triliyun rupiah. Besarnya suramadu dilakukan di Gresik
biaya untuk pembangunan jembatan ini (B) Perubahan cuaca memengaruhi
tentunya menjadi kendala tersendiri bagi pembangunan suatu jembatan
keberlanjutan penyelesaian jembatan ini. (C) Pembiayaan pembangunan
Karena itulah ketika krisis ekonomi jembatan Suramadu sebagian
melanda indonesia, pembangunan menggunakan pinjaman china
jembatan ini sempat tertunda. (D) Krisis ekonomi menghambat
proses penyelesaian
15. Ide pokok yang paling tepat dari pembangunan jembatan
teks di atas adalah …… suramadu
(A) Ikon yang menyaingi menara (E) Suramadu adalah jembatan
Petronas Malaysia terpanjang di Indonesia
(B) Ikon baru Indonesia
(C) Hambatan pembangunan Jembatan 19. Judul yang sesuai dengan bacaan
Suramadu diatas adalah ……
(D) Pembangunan Jembatan Suramadu (A) Jembatan Suramadu
(E) Krisis ekonomi menunda (B) Pembangunan Jembatan
pembangunan jembatan Suramadu Suramadu – Ikon baru Surabaya
dan Madura
16. Pernyataan berikut yang (C) Kerjasama Pembangunan
merupakan fakta dalam dari Jembatan Suramadu dengan
bacaan diatas adalah ……… China
(A) Setiap kota yang memiliki ikon (D) Jembatan Suramadu adalah
tertentu merupakan kota maju Jembatan Terpanjang di
(B) Tidak ada jembatan lain yang Indonesia
mengalahkan jembatan suramadu (E) Hasil Konsorsium Nasional
(C) Pengiriman SBG memerlukan
waktu 120 hari 20. Penyebab penundaan
(D) Sebelum dilakukan pembangunan jembatan Suramadu
pembangunan jembatan adalah ……
suramadu, dilakukan konsorsium (A) Kendala cuaca buruk
terlebih dahulu (B) Kondisi geografis di atas laut
(E) Malaysia adalah negara kerajaan (C) Kondisi geologis yang labil
(D) Mogoknya para pekerja
17. Yang dimaksud Konsorsium (E) Terjadinya krisis keuangan
pada bacaan diatas adalah. . . .
(A) Himpunan beberapa pengusaha
yang mengadakan usaha bersama
(B) Seminar tentang konstruksi dan
proses pembangunan jembatan
(C) Penelitian dan pengembangan
SOAL PENGANTAR 11 (C) Mereka bermain-main di pantai.
(D) Ibu membimbing anaknya
SINTAKSIS-1 berjalan.
(E) Percobaan di laboratorium
berhasil baik.
1.Melaut merupakan pekerjaan rutin 4.Yang menduduki jabatan subjek
bagi nelayan. Pola kalimat di atas kalimat Gedung Sekolah Dasar Inpres
sama dengan pola kalimat… Kampung Baru kebanjiran adalah…
(A) Nelayan berlayar ke tengah laut. (A)Gedung
(B) Mereka bekerja dengan sungguh- (B) Gedung Sekolah Dasar
sungguh. (C) Gedung Sekolah Dasar Inpres
(C) Di tengah laut banyak ikan yang (D) Gedung Sekolah Dasar Inpres
besar. (E) Kampung Baru kebanjiran
(D) Hasil tangkapan dijual di pasar.
(E) Nelayan itu juga berjualan ikan 5.Mangga gedong gincu
di pasar. mengeluarkan aroma yang sangat
harum. Pola kalimat yang sama
dengan pola kalimat di atas terdapat
2.Temuan hari itu belum dapat pada kalimat…
diterapkan di bidang kesehatan. Pola
kalimat tersebut sama dengan pola (A) Lahan-lahannya yang kurang
kalimat… produktif dimanfaatkan sebagai
perkebunan mangga.
(A) Rapat itu memerlukan waktu tiga (B)Negara-negara tujuan ekspor
jam. mangga gedong gincu adalah Jepang,
(B) Belum tentu ada yang mau Eropa, dan Timur Tengah.
mengerjakan pekerjaan seberat itu. (C) Populasi pohon mangga di
(C) Kerusuhan mana pun selalu Indramayu cukup besar.
merugikan masyarakat kecil. (D) Setiap kios menjajakan mangga
(D) Keaiban seseorang tidak boleh gedong gincu berwarna hijau.
dibeberkan di setiap tempat. (E) Aroma mangga gedong gincu
(E) Saya kira belum diketahui oleh tercium pada jarak 2 – 3 meter.
umum.
6. Kurang tuntas penelitian bahaya
3.Perbaikan jalan kereta api sudah narkotika berpola sama dengan
selesai. Kalimat yang sama polanya kalimat…
dengan kalimat tersebut adalah…
(A) Pembangunan di desa ini (A) Polisi sedang menyelidiki
dikerjakan secara gotong royong. penyebab peristiwa itu.
(B) Pedagang itu rajin menawarkan (B) Lagi pula ia masih berduka cita.
barang dagangannya. (C) Belum waktunya kita berbangga
diri.
(D)Masih banyak tugas yang harus ia
selesaikan.
(E) Sangat kosong sorot matanya. (D) Pegawai itu segera mengubah
rencananya setelah bertemu dengan
kepala kantor.
7. Kalimat yang berpola S – P – K (E) Masyarakat tidak hanya
adalah… diingatkan pada kewajibannya, tetapi
(A) Mereka bekerja dengan rajin. juga harus diberi kesempatan untuk
(B) Kondisi pariwisata Lombok memperjuangkan haknya.
tetap menggembirakan.
(C) Pihaknya menerima pembatalan
dua ratus calon wisatawan.
(D) Mereka mengajukan pembatalan 10. Kalimat Paman saya seorang
itu dua hari yang lalu. pelaut terdiri atas sejumlah frasa,
(E) Perlu dikaji lebih mendalam yaitu…
faktorfaktor yang berpengaruh (A) enam frasa
terhadap gejala ini. (B) lima fras
(C) empat frasa
8.Pola kalimat yang sama dengan (D) tiga frasa
Saya mengerjakan ujian ini dengan (E) dua frasa
cermat adalah…

(A) Pertemuan ini diselenggarakan 11. Kata menara dalam frasa


secara sederhana minggu yang lalu Pembangunan menara pemancar
(B) Dia melaporkan hasil kunjungan merupakan unsur…
itu secara lengkap.
(C) Dia datang dengan temannya. (A)pewatas depan
(D) Apa pun hasilnya harus (B) pewatas belakang
dilaporkan dengan lengkap. (C) inti
(E) Beberapa mahasiswa (D) atributif
mempersoalkan kebijakan itu. (E) Koordinatif

9. Pengulangan subjek kalimat dapat


12.Selalu berpakaian sangat
ditemukan dalam kalimat…
sederhana. Inti frasa tersebut adalah...
(A) Peserta seminar yang datang dari
(A) selalu berpakaian
Inggris menginap di hotel ketika
(B) berpakaian sederhana
berada di Jakarta.
(C) Berpakaian
(B) Pengunjung segera (D) Selalu
meninggalkan arena pameran setlah (E) Sederhana
mereka melihatlihat berbagai hasil
kerajinan.

(C) Simposium itu diadakan di


Yogyakarta dan membicarakan
persoalan pemilu.
13.Inti frasa penawaran produk dan
harga adalah…
(A) Penawaran
(B) penawaran produk, penawaran
harga
(C) penawaran produk
(D) produk dan harga
(E) penawaran harga

14.Sambil menikmati salak pondoh


yang rasanya ditanggung manis, para
wisatawan sekaligus dapat menikmati
pemandangan alam. Kalimat inti dari
kalimat di atas berpola sama
dengan…
(A)Sukiman mahasiswa.
(B) Slamet menulis laporan.
(C) Uangnya seribu.
(D) Gino menangis.
(E) Uangnya di Bank.

15. Perbedaan frasa akan mendarat


dengan frasa makan dan minum
adalah bahwa yang pertama berinti
satu, yang kedua berinti dua.
Persamaannya adalah keduanya
merupakan frasa…
(A) verbal

(B) nominal

(C) adjektival

(D) adverbial

(E) Pronominal
SOAL PENGANTAR 12
4. Bagian kalimat yang dicetak
SINTAKSIS-2 miring dari kalimat di bawah ini
adalah adjektiva. Kalimat yang
dimaksud adalah ….
(A) Ibu pergi berbelanja sayur
mayor.
1. Pembangunan yang menyangkut (B) Ibu yang sangat cantik itu
semua aspek kehidupan manusia berbelanja sayur mayor.
harus dilakukan secara merata dan (C) Ibu itu cantik.
berkesinambungan sehingga dapat (D) Ibu bersedih karena anak
meningkatkan kemakmuran dan cantiknya belum pulang.
kesejahteraan masyarakat. Kalimat (E) Ibu ke salon kecantikan untuk
intinya adalah… mempercantik diri.
(A) Pembangunan yang menyangkut
aspek kehidupan.
(B) Semua aspek kehidupan 5. . Penggolongan kata menurut
manusia. Gorys Keraf adalah ….
(C)Pembangunan harus dilakukan. (A) Kata benda, kata ganti, kata
(D)Pembangunan harus merata dan kerja, kata tugas
berkesinambungan. (B) Kata kerja, kata sifat, kata tugas,
(E) Meningkatkan kemakmuran dan kata ganti
kesejahteraan. (C) Kata benda, kata sandang, kata
sifat, kata kerja
2. Stres yang dirasa sangat berat oleh (D) Kata bilangan, kata benda, kata
at menjadi ancaman bagi kesehatan kerja, kata keterangan
orang itu. Berikut ini adalah bagian (E) Kata benda, kata kerja, kata sifat,
inti dari kalimat tersebut… kata tugas.
(A) stres, berat, ancaman
(B) stres, menjadi, ancaman
(C) stres, dirasa, berat 6. Labusel, sebuah kabupaten di
(D) stres, ancaman, kesehatan Sumatera Utara yang sebagian
(E) stres, ancaman, orang daerahnya ditanami kelapa sawit,
ternyata mempunyai potensi alam
3.Hampir semua penumpang yang sangat besar. Kalimat di
tertidur tatkala bus mendekati jalan atas ialah ekspansi dari kalimat ....
yang diselubungi kabut tebal. (A) Labusel sebuah kabupaten di
Unsur-unsur inti kalimat di atas Sumatera Utara
adalah… (B) Labusel mempunyai potensi alam
(A) hampir, semua penumpang (C) Potensi alam yang sangat besar
(B) penumpang, bus (D) Labusel terletak di Sumatera
(C) penumpang, bus, mendaki Utara
(D) bus, mendaki, jalan (E) Sebagian wilayah ditanami
(E) penumpang, tertidur kelapa sawit
7. Sikap negara-negara Barat yang 9. Adik membaca buku.
melarang masyarakatnya
bepergian ke ASEAN, terutama Perluasan dari kalimat di atas yang
ke Indonesia, diKoreksi oleh para polanya masih sama ialah ....
pemimpin ASEAN dengan sebut A. Adik menggunakan kacamata dan
itu sebagai hal yang tak perlu. membaca buku
Inti kalimat di atas ialah .... B. Adik yang menggunakan
(A) Negara Barat tidak perlu kacamata itu, sedang membaca
melarang masyarakatnya ke buku yang engkau pinjamkan.
ASEAN C. Adik saya sedang berguru
(B) Sikap negara Barat ialah hal yang membaca buku yang engkau
tidak perlu pinjamkan
(C) Negara-negara Barat melarang D. Adik sedang membaca sebuah
masyarakatnya ke ASEAN puisi dalam buku itu
(D) Sikap negara-negara Barat Adik meminta abang membaca
diKoreksi oleh pemimpin buku yang dipinjamnya
ASEAN
(E) Sikap negara-negara Barat
melaranag masyarakatnya ke 10. (1) "Dalam penelitian ini, kami
ASEAN telah menunjukkan bahwa kulit di
daerah yang secara alami tanpa
rambut memproduksi inhibitor
yang menghentikan WNT dari
8. Sejak lahirnya konsep aliran gres melakukan tugasnya," ungkap
dalam ilmu kedokteran, yang Profesor Sarah E. (2) Millar
dicetuskan oleh Profesor Linus seorang penulis studi yang
Pauling, yakni ihwal dipublikasikan dalam jurnal “Cell
Ortomolecular medicine yang Reports”. (3) Millar, yang juga
dasarnya ialah studi biologi profesor dermatologi di
molekuler sebagai sains dasar, University of Pennsylvania
penelitian medis diarahkan pada School of Medicine, berkata
molekul-molekul yang secara bahwa WNT sangat penting untuk
normal biologis fisiologis ada perkembangan kantung rambut
dalam badan manusia Inti kalimat dan menyebabkan rambut tumbuh
panjang di atas ialah ... lebih banyak. Inti kalimat pertama
paragraf kedua adalah...
(A) Penelitian medis diarahkan pada
molekul. (A) Penelitian menunjukkan.
(B) Pemikiran gres dalam ilmu (B) Kami menunjukkan.
kedokteran (C) Kulit memproduksi inhibitor.
(C) Ortomoleculer medicine sebagai (D) Profesor Sarah E Millar
sains dasar mengungkapkan pernyataan
(D) Biologi molekuler ialah sains (E) Pernyataan penulis studi yang
dasar dipublikasikan dalam jurnal
(E) Pemikiran gres dicetuskan oleh “Cell Reports”.
Profesor Linus Pauling
(E)Petugas DLLAJ dan masyarakat
11. 1) Setiap tanggal 12 Oktober, setempat membantu membersihkan
rakyat Amerika Serikat jalan dan mengatur kendaraan.
merayakan Hari Kolumbus. (2)
Namun, tak semuanya ingin
mengetahui kisah sejarah palsu 13.Kakak membelikan ayah mobil
dan berdarah itu. (3) Pasalnya, mini buatan Jepang pekan lalu. Yang
Kolumbus yang dicatat dalam menduduki fungsi keterangan dalam
sejarah sebagai penemu Benua kalimat diatas adalah…
Amerika ternyata tak pernah (A) Ayah; mobil mini
menginjakkan kaki di Amerika (B) Buatan Jepang; pekan lalu
Utara. (4) Bahkan, menurut para (C) Ayah; pekan lalu
sejarawan termasuk Laurence (D) Mobil mini buatan Jepang
Bergreen yang menulis buku (E) Pekan lalu
Kolumbus: The Four Voyages ,
ada fakta bahwa Kolumbus kejam 14. Pembangunan KUD ini kami
terhadap orang-orang lokal di memperoleh banyak bantuan
wilayah yang ia taklukkan. (5) pemerintah, baik berupa bibit dan
Kolumbus menerapkan sistem pupuk maupun peralatan pertanian.
encomienda , yaitu program Untuk memperjelas ide yang
memperbudak pihak yang terjajah terkandung dalam kalimat di atas,
dan telah dilaksanakan sejak bagian kalimat Pembangunan KUD
Eropa abad pertengahan. Apakah ini harus diganti menjadi…
inti kalimat (3)? (A) KUD yang dibangun ini…
(B) Membangun KUD ini…
(A) Kolumbus dicatat sebagai (C) Bangunan KUD ini…
penemu Benua Amerika, (D) Untuk membangun KUD ini…
(B) Kolumbus tak pernah (E) Dibangunnya KUD ini…
menginjakkan kaki
(C) Kolumbus tercatat tak pernah 15. Ilmu merupakan sebuah modal
menginjakkan kaki. hidup yang dapat dipanen sepanjang
(D) Kolumbus terkenal dalam masa sehingga untuk mencari ilmu
sejarah. adalah sebuah kewajiban manusia
(E) Kolumbus penemu Benua agar dapat menjalani kehidupan
Amerika. dengan baik. Kalimat tersebut akan
menjadi baik bila diperbaiki dengan
12. Kalimat inti pada kalimat Antrean cara…
Kendaraan perlahan baru dapat (A) menambahkan kata adalah
diatasi setelah petugas DLLAJ sebelum kata merupakan
dan masyarakat setempat (B) mengganti kata sehingga dengan
membantu membersihkan jalan akibatnya
dan mengatur kendaraan adalah… (C) menambahkan tanda koma (,)
(A) Antrean kendaraan baru. sebelum kata sehingga
(B) Antrean kendaraan (D) menghilangkan kata untuk
diatasi, (E) mengganti kata dengan menjadi
(C) Antrean kendaraan diatasi secara
petugas DLLAJ.
(D) Antrean kendaraan
diatasi petugas DLLAJ dan
masyarakat.
SOAL PENGANTAR 13 4.Dengan kelemahanlembutannya,
Soni berhasil mengatasi kemarahan
SINTAKSIS-3 orang itu. Berdasarkan jumlah
klausanya, kalimat di atas sama
1. Bentuk aktif yang tepat dari dengan…
kalimat Hal tersebut sudah saya (A)Saya akan datang bila diminta.
kemukakan kepada Saudara adalah… (B)Karena terlambat, dia dilarang
masuk.
(A)Sudah saya kemukakan hal (C)Kebersihan itu sebagian dari
tersebut kepada Saudara. iman.
(B) Hal tersebut sudah dikemukakan (D)Jika dia datang, segera
oleh saya kepada saudara. berikan surat ini kepadanya.
(C) Saya sudah kemukakan hal (E)Risti membersihkan kamar, Risti
tersebut kepada saudara. halaman.
(D) Saya kemukakan hal tersebut
kepada saudara. 5. Penggabungan klausa Peristiwa
(E) Saya sudah mengemukakan itu terjadi dengan klaus. Keluargaku
hal tersebut kepada Saudara. sedang dalam suasana berkabung
akan membentuk kalimat majemuk
2. Rumah besar itu terbakar tadi bertingkat yang menyatakan
malam. Pola kalimat tersebut sama hubungan…
dengan pola kalimat… (A) Waktu
(A) Pemerintah menyerahkan (B) Syarat
Upakarti kepada perajin yang berjasa. (C) Tujuan
(B) Pedagang itu menawarkan (D) Penjumlahan
barang dagangannya. (E) Sebab
(C) Dia memperhatikan polisi
mengatur lalu lintas.
(D) Kami berteduh di bawah 6. (1) Kami akan membahas masalah
pohon. itu sekali lagi. (2) Masalah itu dapat
(E) Berita itu disiarkan ke dimengerti. Kedua kalimat di atas
seluruh negeri. dapat dihubungkan dengan
menggunakan konjungsi
subordinatif penunjuk hubungan…
3.Andi, paman Dila, amat baik (A) Tujuan
dengan tetangganya. Inti kalimat di (B) Cara
atas adalah… (C) Waktu
(A) Andi amat baik. (D) Sebab
(B) Paman Dila amat baik. (E) akibat
(C) Andi baik dengan tetangganya.
(D) Andi, paman Dila, baik dengan
Tetangganya 7. Jaminan dan perlindungan hukum
(E) Andi baik. terhadap hak-hak asasi warga negara
yang secara tegas diatur di dalamnya
. harus mendapat penghormatan yang
setinggi-tingginya. Kalimat di atas
mempunyai pola dasar yang sama
dengan…
(A) Ibu seorang wiraswasta.
(B) Adik berlari kencang.
(C) Pengemis itu sangat malas. 13. Cermatilah paragraf di bawah ini!
(D) Pengacara itu membela kliennya.
(E) Perampok itu telah diciduk oleh Bencana yang datang beruntun akhir-
petugas keamanan. akhir ini ... syok. Namun demikian,
janganpanik. Ketenangan jiwa ...
dalam menghadapi situasi darurat
8. Dalam setahun, luas hutan rakyat seperti ini. Segera kumpulkansemua
yang ditebang itu sekitar 1.400 anggota keluarga atau ketahui
hektare. Kalimat diatas berpola… keberadaan lokasi anggota keluarga
(A) S + P yang belumberkumpul.
(B) S + P + O Frasa yang tepat untuk melengkapi
(C) S + P + O + K paragraf rumpang tersebut adalah .
(D) K + S + P (A) akan membuat, sering
(E) K + S + P + O dibutuhkan
(B) sering membuat, sangat
dibutuhkan
9. Semua, sawah ladangnya, sudah (C) telah membuat, akan dibutuhkan
terjual. Bentuk frasa sawah ladangnya (D) selalu membuat, akan dibutuhkan
sama dengan bentuk frasa dalam… (E) sudah membuat, selalu
(A) Kami naik kereta api cepat ke dibutuhkan2.
Surabaya.
(B) Besi berani ini terbuat dari baja 14. Cermatilah kalimat rumpang
pilihan. berikut!
(C) Dia mengusap mukanya dengan
saputangan putih. ..., lampu-lampu di bandara padam
(D) Dia menyalurkan hobi memasak lalu para pembajak mulai
dengan mendirikan rumah makan. melancarkan tembakan.
(E) Anak istrinya ikut menunjang Klausa yang tepat untuk melengkapi
kariernya. kalimat majemuk campuran tersebut
adalah ....
10. Kalimat Ayah kami sedang (A) Penyerbuan itu terjadi
membaca buku baru terdiri atas (B) Sebelum penyerbuan itu terjadi
sejumlah frasa, yaitu… (C) Setelah penyerbuan itu terjadi
(A) enam frasa (D) Para pembajak menaiki pesawat
(B) lima frasa (E) Karena para pembajak menaiki
(C) empat frasa pesawat
(D) tiga frasa
(E) dua frasa

11. Semua buku itu akan saya baca.


Kata buku pada frasa semua buku itu
merupakan unsur…
(A) pewatas depan
(B) pewatas belakang
(C) inti
(D) atributif
(E) koordinatif
15. Cermatilah paragraf berikut!
(1) Masyarakat tengah menikmati
kebebasan membaca buku.
(2) Apalagi didukungoleh keberadaan
buku yang terbit seperti jamur di
musim hujan. (3) Secara logika, saat
iniseharusnya tumbuh semangat dan
budaya membaca yang semakin kuat.
(4) Namun,tampaknya hal tersebut
belum memberi banyak perubahan
karena berbagai faktor, sepertidaya
beli masyarakat yang rendah. (5) Di
tengah kebutuhan hidup yang
semakin tinggi,masyarakat belum
sempat berpikir untuk menjadikan
buku sebagai bagian dari
menubelanjanya.
Kalimat berpola SPO dalam paragraf
tersebut terdapat pada nomor ....
A. (1)
B. (2)
C.(3)
D.(4)
E.(5)
SOAL PENGANTAR 14 4. Penulisan bilangan yang tidak
sesuai dengan yang digariskan
PUEBI 1 dalam EYD dijumpai dalam
kalimat...
(A)Dari hasil penelitian yang
1. Penulisan akronim yang salah dilakukan Yayasan Kusuma Buana
dibawah ini adalah ... di sekolah-sekolah di Jakarta,
(A) Posyandu prevalensi anemia disetiap sekolah
(B) SIM antara 20 persen dan 35
(C) Bulog persen,bahkan ada yang mencapai
(D) SISKAMLING 60 persen.
(E) pemilu (B)Sekolah Menengah Kejuruan
Negeri (SMKN) X, dua bulan yang
2. Penulisan partikel di bawah ini lalu, dikunjungi oleh 20 guru dan 5
yang benar adalah ... pejabat Pemerintah Malaysia.
(A) Baca lah buku itu baik-baik. (C) Penghargaan Festival Teater leh
(B) Siapa kah yang menjamin masa 15 finalis yang tampil di Teater
depanmu? Luwes Institut Kesenian Jakarta dan
(C) Apapun yang dimakannya, ia Teater Studio Taman Iskandar
tetap kurus. Marzuki.
(D) Jangankan dua kali, satu kalipun (D) Dari 422 laporan yang masuk di
kamu belum pernah bermain ke Komisi Kejaksaan, sebanyak 202
rumahku. laporan dilanjutkan ke Jaksa Agung.
(E) Walaupun ia sudah lulus di PTN, (E)Dari tiga ratus laporan
ia tetap rendah hati. masyarakat yang masuk ke Komisi
Kejaksaan pada kurun
3. Berikut ini adalah penulisan April–Desember 2006, yang
kalimat yang sesuai dengan Pedoman diteruskan ke Jaksa Agung sebanyak
Umum Ejaan Bahasa Indonesia yang dua ratus laporan.
benar, kecuali...
(A) Dalam bahasa Minang, terdapat
kata ‘mangicuah’, artinya berbohong. 5. Penulisan gabungan kata di bawah
(B) Calon jemaah haji akan ini yang benar adalah...
diberangkatkan hari ini ke Mekah.
(C) Selama 350 tahun bangsa (A) Maha Kuasa
Indonesia dijajah oleh Belanda. (B) Mahaesa
(D) Di Indonesia, Presiden langsung (C) Kosa Kata
dipilih oleh rakyat.
(D) Jajarangenjang
(E) Bawang Daun menjawab, “Aku
tak sudi mengotori tanganku!”. (E) Antipenuaan

6. 1) Penggunaan istilah Empat


Pilar pada hakekatnya ingin
meneguhkan Pancasila sebagai
dasar berbangsa dan bernegara.
(2) Pancasila tetaplah dasar negara
meski pada saat bersamaan
dilekatkan dengan istilah semisal
Pancasila sebagai pandangan hidup
bangsa, Pancasila adalah ambulans, mobil pemadam
perjanjian luhur bangsa, dan lain kebakaran, dan mobil pengangkut
sebagainya. (3) Bung Karnopun sampah
sebagai penggali Pancasila pernah
memaknainya sebagai bintang
penuntun arah perjuangan bangsa.
(4) Ini menunjukkan terdapat 7. Dalam bacaan di atas dijumpai
betapa banyak keragaman fungsi pemakaian tanda koma (,) yang
atau peran Pancasila bagi bangsa dan tidak tepat pada ....
negara kita. (5) Keragaman makna (A) kalimat (1)
seperti itu tentu tak akan (B) kalimat (2)
menggoyahkan kedudukan (C) kalimat (3)
Pancasila sebagai dasar negara. (6) (D) kalimat (4)
Wacana Pancasila sebagai dasar atau (E) kalimat (5)
bukan itu sudah lama usai. (7) Kita
jangan lagi kembali ke belakang..
Kesalahan penggunaan ejaan 8. Dalam bacaan di atas terdapat
ditemukan pada kalimat .... penulisan kata yang salah,
yaitu ...
(A) 1 dan 2 (A) bersubsidi (kalimat 1)
(B) 3 dan 6 (B) kilo liter (kalimat 2)
(C) 1 dan 3 (C) kuota (kalimat 3)
(D) 3 dan 5 (D) Peraturan Menteri (kalimat 4)
(E) 4 dan 5 (E) ambulans (kalimat 5)

9. Manakah penulisan singkatan dan


Bacalah teks di bawah ini untuk akronim yang penulisannya tepat?
menjawab nomor 7 dan 8!
(A) UUD, Pemilu, s/d
1) Kelangkaan bahan bakar (B) UPI, Unpad, Osis
minyak berawal dari niat (C) Puskesmas, a.n. , Rp.
pemerintah mengendalikan (D) LIPI, Unpad, tilang
penggunaan BBM bersubsidi. (E) OSIS, Ikapi (Ikatan Penerbit
(2) Tahun lalu, pemakaian BBM Indonesia), daring (dalam jaringan)
bersubsidi naik dari kuota yang
ditetapkan 40 juta kilo liter
menjadi 45,27 juta kilo liter. (3)
10. Manakah kelompok kata yang sesuai
Jika tidak dikendalikan, kuota
diperkirakan akan terlampaui dengan kaidah PUEBI ?
dan berakibat semakin (A) sukarela, antar warga
memberatkan APBN. (4)Niat
(B) mempunyai, memerhatikan
pemerintah mengendalikan
(C) dukacita, semi final
penggunaan BBM bersubsidi,
(D) antar-jemput, tuna asmara
dituangkan dalam Peraturan
(E) non-Indonesia, saputangan
Menteri ESDM nomor 1 Tahun
2013 tanggal 2 Januari 2013. (5)
Di dalamnya diatur, kendaraan
dinas tidak boleh memakai BBM
bersubsidi, kecuali mobil
Bacalah teks di bawah ini untuk termasuk Kepler 62f yang berpotensi
menjawab nomor 11 s.d. 15! dihuni.

Penemuan Langka Berupa Planet 11.


Mirip Bumi di Dekat Pusat Galaksi
(A) TIDAK PERLU DIPERBAIKI
Ilmuwan dan (B) astronaut
(11) angkasawan kembali mengungkap (C) astronom
sebuah temuan baru di dekat pusat (D) antariksawan
galaksi yang disinyalir sebuah planet (E) geofisikawan
mirip dengan bumi. Penemuan ini
diungkap para ilmuwan dari University
of Canterbury (UC) yang memberi
istilah (12) dengan planet baru itu
dengan nama 'Super Earth'. 12.
Penemuan ini terbilang (A) TIDAK PERLU DIPERBAIKI
sangat langka, seperti yang diungkapkan (B) ke
ilmuwan Antonio Herrera Martin. Hal (C) Dari
itu karena planet ini diklaim menjadi (D) pada
satu-satunya planet yang memiliki (E) di
ukuran dan juga orbit serupa dengan
bumi.
Para ilmuwan sebelumnya 13.
telah mengatakan bahwa Mars adalah (A) TIDAK PERLU DIPERBAIKI
salah satu planet yang layak dihuni oleh (B) terpublikasi
manusia. Namun, kini ada penemuan (C) terpublikasikan
baru yang juga terdapat planet mirip (D) pempublikasian
dengan Bumi. Benda langit itu juga (E) Memublikasikan
diyakini memiliki orbit antara Venus dan
Bumi yang mengorbit bintang induknya.
Planet ini berada nyaris di tengah galaksi 14.
Bima Sakti. (A) TIDAK PERLU DIPERBAIKI
Dalam penelitian yang (B) teknik
telah (13) dipublikasikan di sebuah (C) teknic
jurnal ilmiah ini disebutkan satu hari di (D) teknis
planet ini setara dengan 617 hari, (E) tehnik
sedangkan di Bumi hanya 365 hari.
Untuk menemukan planet serupa bumi
ini, peneliti UC menggunakan
(14) technik microlensing gravitational. 15.

Microlensing pertama kali (A) TIDAK PERLU DIPERBAIKI


diterapkan secara (15) in dependend (B) in-dependen
pada tahun 2018 oleh Eksperimen (C) in dependen
Gravitasi Lensa Optik (OGLE) (D) Independen
menggunakan teleskop di Chili. NASA (E) independent
sendiri telah mengumpulkan beberapa
penemuan planet mirip bumi yang
menarik dalam beberapa tahun terakhir,
Bacalah teks di bawah ini untuk 19.
menjawab nomor 16 s.d. 20!
(A) TIDAK PERLU DIPERBAIKI
(B) 6-8 persen
(C) enam sd delapan persen
(D) enam sd delapan %
Harga karet alam di pasar dunia
(E) enam sampai dengan delapan
(1)Tahun 2013 tetap bagus. Seiring
persen
meningkatnya produksi kendaraan,
permintaan terhadap karet meningkat.
Menurut (2)Chairman gabungan Perusahaan
Karet Indonesia, (3)Asril Sutan Amir, 20.
tingginya permintaan mendorong
(A) TIDAK PERLU DIPERBAIKI
peningkatan laju ekspor karet. Ekspor karer
(B) Oktober
tahun 2013 diperkirakan meningkat (4)enam
(C) October
sampai dengan delapan persen dari total
(D) October
ekspor 2012 atau mencapai 2,4 juta ton.
(E) October
Ekspor karet tahun 2012 hingga (5)oktober
adalah sebanyak 2,01 juta ton. Hingga akhir .
2012 ekspor karet bertambah (6)400.000
ton.

16.

(A) TIDAK PERLU DIPERBAIKI


(B) tahun 2013
(C) “tahun 2013”
(D) tahun 2013
(E) “Tahun 2013”

17.

(A) TIDAK PERLU DIPERBAIKI


(B) Ketua
(C) Ketua
(D) Chairman
(E) “chairman”

18

(A) .TIDAK PERLU DIPERBAIKI


(B) Asril Sutan amir
(C) Asril Sutan Amir
(D) Asril Sutan Amir
(E) “Asril Sutan Amir”
SOAL PENGANTAR 15 4.Pemakaian tanda garis miring (/)
sesuai dengan kaidah, kecuali...
PUEBI 2 (A)Paketnya dapat dikirim lewat
laut/ darat.
(B)Di amplop itu tertera d/a Pak
1. Penulisan kata bilangan yang Hilmi.
tidak tepat terdapat pada... (C)Dia tinggal di Blok AA I/14.
(A)Karya sastra STA ini telah (D)Pekerja itu hanya mendapat
dicetak lebih dari 20 (dua puluh) kali. gaji sebesar Rp200.000,00/ bulan.
(B)Ketujuh belas orang itu (E)Balai pertemuan itu dibangun
bekerja bersama-sama. dengan dana anggaran tahun
(C)Tahun ini Indonesia akan 2004/2005.
berulang tahun ke-57
kemerdekaannya. 5.Pemakaian ejaan yang benar
(D)Dialah orang pertama dalam terdapat dalam kalimat...
perusahaan itu. (A)20 orang korban pembajakan
(E)Hanya dua pertiga bagian meninggal.
dari pekerjaannya selesai. (B)25 orang korban pembajakan
meninggal.
2. Penggunaan tanda baca yang (C)Dua puluh orang korban
tidakdapat dibenarkan terdapat dalam pembajakan meninggal.
kalimat... (D)Korban pembajakan sebanyak
(A)Sejak masa reformasi sampai dua puluh lima orang meninggal.
kini, aksi demonstrasi hampir (E)Korban pembajakan sebanyak
menjadi tontonan harian masyarakat. 20 orang meninggal.
(B)Di samping Jakarta, kota yang
juga mengalami kerusuhan adalah 2. Tanda titik tidak dipakai pada...
Medan, Solo, dan Makassar. (A)akhir singkatan nama orang
(C)Seorang pengamat mengatakan, (B)akhir singkatan gelar, jabatan,
“Sampai beberapa waktu ke depan pangkat, dan sapaan
kerusuhan tampaknya masih (C)singkatan kata atau ungkapan
akan terjadi.” yang sudah sangat umum
(D)Bila tetap mengutamakan hawa (D)singkatan yang berupa akronim
nafsu dan rasa dengki, reformasi (E)penulisan angka jam, menit, dan
yang diharapkan, tidak hanya detik yang menunjukkan waktu
menjadi slogan kosong belaka.
(E)Akan tetapi, dengan kedewasaan 7.Penggunaan tanda koma yang tepat
dan kejernihan kita masing-masing, terdapat pada kalimat...
semua masalah dapat kita selesaikan. (A)Dia lupa akna janjinya, karena
sibuk.
3.Pemakaian singkatan yang benar (B)Anak itu berpendapat, bahwa soal
adalah... itu tidak penting.
(A)s/d (sampai dengan) (C)Anak itu malas, sehingga tidak
(B)s.d.a (sama dengan di atas) naik kelas.
(C)ub. (untuk beliau) (D)Saya tidak akan datang, kalau hari
(D)Tbk. (terbuka) hujan.
(E)d-a (dengan alamat) (E)Saya ingin datang, tetapi hari
hujan.
8.Penggunaan tanda koma dalam 11.Penulisan singkatan sarjana sastra
kalimat berikut benar, kecuali... di belakang nama diri, menggunakan
(A)Semuanya terdiam, ketika bunyi tanda baca sebagai berikut...
itu terdengar untuk ketiga kalinya. (A)Prof. Dr. Prayongki S.S
(B)Tatkala mendengar hasil yang (B)Prof. DR. Prayongki, S.S.
menggembirakan itu, mereka (C)Prof. D.R. Prayongki, SS
mengucapkan syukur. (D)Prof. Dr. Prayongki, S.S.
(C)Dalam menolak pendapat (E)Prof. Dr. Prayongki S.S
seseorang di sebuah forum diskusi,
kita hendaklah menyampaikannya 12.Penulisan gabungan kata yang
dengan sopan. benar adalah...
(D)Sebagai seorang anak,kita harus (A)tanggungjawab
berbakti kepada orang tua. (B)sumberdaya
(E)Oleh sebab itu, pertanyaan Anda (C)antarkota
tidak relevan dengan masalah yang (D)anaksungai
sedang dibahas. (E)sebarluas

9.Penggunaan tanda baca yang


mengikuti EYD terdapat pada 13. Penulisan judul yang sesuai
kalimat... dengan EBI adalah….
(A)Lukisan paling tua dibuat pada (A) Gerak-Gerik Pencuri di Malam
tahun 2002; dan yang terbaru pada Hari dengan Alat Saktinya
tahun2005. (B) Kecil-kecil Jadi Pengusaha Buah
(B)Kalau tetangga bersikap tak acuh, Mangga
kelinci kami akan mati terpanggang. (C) Berjalan-Jalan Ke Kota
(C)Di sana ada harimau loreng, Kembang Bandung
penguin dan gorila. (D) Si Pohan dan Si Juna Pemburu
(D)“Kamu berdiri di sana!,” katanya. Harta Karun di Maribaya
(E)Kami semua akan berangkat, (E) Surat Cinta Dari Pengagum
minggu ini. Rahasia

10.Penggunaan tanda baca yang tidak 14.Penulisan huruf kapital di bawah


tepat terdapat pada kalimat... ini yang salah adalah….
(A)Fakultas sastra itu terdiri atas dua (A) Seharusnya dia berpikir dan
jurusan: Sastra Indonesia dan Sastra berdoa terhadap Tuhan Yang
Inggris. Maha Kuasa.
(B)Akan tetapi, kami tidak sanggup (B) Pria itu dikejar-kejar pihak
menanganinya. berwenang karena mengaku
(C)Ia tidak akan ke pertemuan itu, sebagai nabi.
meskipun diundang. (C) Bulan ini ayah saya akan dilantik
(D)Alhamdulillah, kita bisa menjadi menteri.
menuntaskan pekerjaan ini. (D) Hal yang kurang menyenangkan
(E)walaupun banyak yang ketika ia diketahui Suku Sunda.
menentang, rancangan undang- (E) Perang Kemerdekaan menjadi
undang tersebut tetap akan disahkan. saksi rakyat Indonesia yang tak
terlupakan
Bacalah teks di bawah ini untuk 17.
menjawab nomor 15 s.d. 18!
(A) TIDAK PERLU DIPERBAIKI
(B) Sang
Para peserta diberitahu tentang (C) Sang
pilihan dokter untuk menawarkan opsi (D) Sang
15 bypass jantung. Opsi ini berpeluang (E) ‘sang’
menjaga kesehatan pasien selama
bertahun-tahun, meski ada kemungkinan
kematian saat operasi. Mungkin seperti
18.
sudah diprediksi, para peserta menilai
keputusan 16 sang dokter jauh lebih (A) TIDAK PERLU DIPERBAIKI
keras jika mereka diberi tahu bahwa (B) non kooperatif
pasien meninggal setelah operasi (C) non-kooperatif
dibandingkan ketika mereka diberitahu (D) non-Kooperatif
bahwa pasien bertahan hidup meskipun (E) nonkooperatif
manfaat dan 17 resikonya sama persis
dalam setiap kasus. Bagaimanapun,
begitu Anda mengetahui tentang tragedi 19. Kalimat yang betul seluruh
itu, sulit untuk tidak menyalahkan, ejannya adalah...
dokter membuat para peserta (A)Belokkan sepedamu pada
mempertanyakan kompetensinya dan 18 belokan pertama!
nonkooperatif. (B)Belokan sepedamu pada belokan
pertama!
(C)Belokkan sepedamu pada
belokkan pertama!
15.
(D)Belokan sepedamu pada belokan
(A) TIDAK PERLU DIPERBAIKI pertama!
(B) Bypass (E)Belokkan sepedamu pada
(C) “bypass” belokkan pertama!
(D) bypass
(E) “bypass” 20.Penulisan kalimat yang sesuai
dengan EYD adalah...
(A)Bagi Ridha, tes C.P.N.S tahun
ini merupakan kesempatan yang
16 kedua kalinya.
(B)Karena tidak mampu bersaing,
(A) TIDAK PERLU DIPERBAIK
toko itu gulung tikar.
(B) Risiko
(C)Pada tahun 1998, Indonesia
(C) Risikonya
memasuki masa Reformasi.
(D) Risikonya
(D)Ketulusan hati pengarang itu
(E) risiko
sudah tersebarluas.
(E)SULUK adalah sebuah karya
sastra dalam Islam
SOAL PENGANTAR 16 Berjalan (6) kearah selatan melalui
jalan aspal mulus, sekitar setengah jam
PUEBI 3 (kurang lebih 12 kilometer), kita akan
sampai di Benteng Anoi Itam. Benteng
ini lokasinya sangat strategis. Letaknya
Lautan Sabang adalah pintu di puncak tebing tepi pantai, menghadap
masuk ke Selat Malaka. Tampak jauh di ke mulut Selat Malaka dari arah Laut
lepas laut, sesekali kapal tanker atau Andaman.
kapal bermuatan kontainer lewat. Bisa
dibayangkan, dulu saat Perang Dunia 2 Ada beberapa  (4) bunker tempat
yang lewat adalah kapal-kapal perang menyimpan amunisi dan sebuah bunker
Amerika dan sekutunya. Meriam jenis atau benteng pertahanan dengan sebuah
coastal defense milik Jepang tersebut meriam (5) coastal defense. Sayangnya,
menembakan peluru ke arah kapal-kapal meriam itu sudah tidak utuh, hanya
itu agar mereka tak memasuki Selat tinggal larasnya saja. Untuk mencapai
Malaka sebagai pintu masuk kawasan tempat ini, kita harus naik tangga.
Asia Tenggara, Kawasan yang di akhir Bunker pertama ada di sebelah kanan
Perang Dunia 2 dikuasai Jepang. tangga.

Sejarah Sabang tak bisa terlepas


dari sejarah kolonial Portugis, Belanda,
hingga Jepang di Perang Dunia 2. Pada 1. Kata bentukan yang salah pada
12 Maret 1942 pukul 00:00, dengan paragraf pertama terdapat pada
sandi “Operation T”, satu batalyon ….
Divisi Darat Kobayashi Kekaisaran
Jepang, mendarat di Sabang. Dalam (A) Sesekali
kurun waktu 1942 – 1945 Sabang
menjadi pangkalan angkatan laut (B) Bermuatan
Jepang yang besar untuk menghadapi
sekutu. Tak heran jika di Pulau Weh (C) Menembakan
banyak sekali peninggalan militer
Jepang. Ada bunker-bunker yang (D) Memasuki
sebagian masih bisa dikunjungi, bekas
benteng, bahkan bekas lokasi (E) Dikuasai
pembantaian yang memilukan.
2. Pada kalimat bercetak tebal,
(1) Beberapa bunker dan benteng tanda koma seharusnya
yang masih terawat dan mudah bisa dituliskan pada ….
dikunjungi adalah BentengAnoi Itam di (A) Sebelum kata Sabang
Kawasan Anoi Itam dan bekas benteng (B) Sebelum kata yang
di kawasan Sabang Fair. (2) Sabang Fair (C) Sebelum kata untuk
tepat berada di pinggir pantai yang (D) Sebelum dan sesudah
menghadap ke mulut Teluk Sabang, sisi kata Jepang
utara Pulau Weh. (3) Di deretan Pantai (E) Sesudah kata waktu dan sebelum
itu, tepatnya di sisi sebelah kiri ada kata Sabang
Pelabuhan Sabang. (4) Ada bekas
ruang meriam dan amunisi. (5)
Sayangnya, laras-laras meriam itu sudah
tidak berada di bunkernya, tapi
dipindahkan ke halaman Sabang Fair.
6.

(A) TIDAK PERLU DIPERBAIKI


(B) ke-arah
3. Penulisan huruf kapital tidak (C) arah
tepat pada paragraf ketiga (D) ke arah
terdapat pada kalimat nomor …. (E) ke

(A) (1)

(B) (2)

(C) (3) 7. Secara berurutan kata-kata


berikut yang merupakan hasil
(D) (4) proses afiksasi, reduplikasi, dan
kompositum adalah ….
(E) (5)
(A) Berlari, memukul-mukul, banting
Pertimbangkan apakah kata atau harga
kalimat pada setiap nomor bercetak (B) Berpakaian, lauk-pauk, sayur-
tebal TIDAK PERLU DIPERBAIKI mayur
(A) atau diganti dengan pilihan lain (C) Berhadap-hadapan, tetumbuhan,
yang tersedia (B, C, D, atau E) untuk rumah sakit
mengerjakan soal nomor 4 – 6! (D) Memperbolehkan, kura-kura,
sapu tangan
4.
(E) Bersenda gurau, dedaunan, lelaki
(A) TIDAK PERLU DIPERBAIKI

(B) “bunker” 8. Kalimat yang memiliki


(C) bungker penggunaan tanda koma yang tepat
adalah …
(D) Bunker
A. Setibanya di Kebonjahe yang
sejuk dan tidak terlalu ramai, kami
(E) “bungker”
berjalan-jalan di sepanjang jalan
utama Kota Kebonjahe.
B. Aku pun tidak membukakan pintu
saat dia mengetuk pintu, karena aku
5.
yakin adikku atau ayahku akan
membukakan pintu itu.
(A) TIDAK PERLU DIPERBAIKI
C. Kereta tak selalu berjalan stabil,
(B) coastal defense
adakalanya kereta berjalan pelan dan
(C) Coastal Defense
kadang-kadang juga kereta berjalan
(D) “coastal defense”
cepat.
(E) koastal defen
D. Kesabarannya hilang, sehingga
dia langsung mematikan telepon
genggamnya.
E. Peraturan di kantor lama, ternyata (E) Beberapa tokoh anti rasisme dari
tidak berlaku di kantor baru. beberapa negara sepakat untuk
menanda tangani suatu
kesepahaman.

11. Bentuk kata serapan yang benar


terdapat dalam kalimat…

(A) Radio pada mobil merupakan


asesoris yang digemari oleh
banyak konsumen

(B) Tarian itu dianggap akseptebel


untuk ditampilkan di depan tamu
9. Penurunan cukai impor beras
menjadi Rp. 450,- per kilogram,
per 1 Januari 2008 dimaksudkan
(C) Pada jaman penjajahan di kota
agar Bulog dapat mengendalikan Malang terbit banyak koran,
harga beras . Kalimat tersebut koran-koran tersebut tidak dapat
akan benar ejaannya apabila... terbit secara kontinyu
(UM UGM 08)
(A) Rp.450,- ditulis Rp 450,00. (D) Pematuhan terhadap prinsip kerja
(B) per kilogram ditulis perkilogram. sama dan prinsip kesopanan akan
melahirkan komunikasi yang
(C) per 1 januari 2008 ditulis per-1
bonafide sekaligus melahirkan
januari 2008. pula wacana yang wajar
(D) impor ditulis import.
(E) sebelum kata agar ditambahkan (E) Komunikasi merupakan syarat
tanda koma (,). esensial bagi terciptanya
prasyarat tumbuhnya kegiatan
10. Kalimat yang penulisannya yang terbuka, yang tidak elit.
mengikuti EYD adalah
12. Penerapan EYD yang benar
(A) Sepuluh tahun yang lalu terdapat dalam kalimat….
Keluarga Satrio tinggal di jalan
RE Martadinata, No.5. (A) Kegiatan perkuliahan
(B) Sebagian besar dari kita dimulai,semua kewajibanpun
beranggapan bahwa kebudayaan administrasi harus sudah
Timur lebih halus jika diselesaikan
dibandingkan dangan
kebudayaan Barat. (B) H.Krupper, seorang ahli pra
(C) Ia mengakui ke dalaman sejarah , melaporkan bahwa telah
wawasan tokoh masyarakat ditemukan situs Bukit Karang di
kampung Naga itu. daerah Langsa
(D) Pengusaha besar itu menghadiahi
atlit-atlit berprestasi internasional (C) Akhirnya, Diana mendapatkan
dangan bea siswa untuk belajar izin belajar keluar negeri setelah
di luar negeri. berjuang bertahun-tahun
(D) Hari jum’at yang lalu, hujan 15. Penulisan bilangan yang tidak
deras mengguyur Jakarta sesuai dengan yang digariskan
seharian dalam EYD dijumpai dalam
kalimat
(E) Kami tidak bisa menerima surat
tersebut, karena faksimile di (A) Dari hasil penelitian yang
kantor kami sedang rusak dilakukan Yayasan Kusuma
Buana di sekolah-sekolah di
Jakarta, prevalensi anemia
disetiap sekolah antara 20 persen
13. .Penulisan kata serapan dalam dan 35 persen,bahkan ada yang
kalimat-kalimat berikut ini benar, mencapai 60 persen.
KECUALI yang terdapat pada
kalimat (B) Sekolah Menengah Kejuruan
Negeri (SMKN) X, dua bulan
(A) Setiap pagi kumandang azan itu yang lalu, dikunjungi oleh 20
membangunkan saya. guru dan 5 pejabat Pemerintah
Malaysia
(B) Muna bekerja di laboratorium itu
sebagai bakteriolog (C) Penghargaan Festival Teater
diperebutkan oleh 15 finalis yang
(C) Orang yang mengusahakan bank tampil di Teater Luwes Institut
disebut bankir Kesenian Jakarta dan Teater
Studio Taman Iskandar Marzuki
(D) Keamanan di ibukota merupakan
barometer keamanan di
Indonesia

(E) Sebelum meninggal, N sempat


menyelesaikan otobiografinya

14. Kalimat berikut yang seluruh (D) Dari 422 laporan yang masuk di
ejaannya ditulis sesuai dengan Komisi Kejaksaan, sebanyak 202
aturan EYD adalah laporan dilanjutkan ke Jaksa
Agung
(A) Tiga per empat bagian diberikan
kepada orang lain

(B) Sekalipun dia belum pernah ke (E) Dari tiga ratus laporan
Bali masyarakat yang masuk ke
Komisi Kejaksaan pada kurun
(C) Kemana pun perginya, ia tidak April – Desember 2006, yang
lupa membawa kamera diteruskan ke Jaksa Agung
sebanyak dua ratus laporan
(D) Kalaupun hari itu turun hujan
lebat, saya tetap bersedia datang
pada acara Anda

(E) Mereka satu per satu datangnya


tetapi tidak satu pun yang absen
(C) Barang-barang itu perlu
dikelompokkan dan diberkas
dalam satu kotak karton tebal,
agar memudahkan dalam
pengecekannya.

(D) Sebagai akibat dari kurangnya


pemahaman terhadap topik yang
dibicarakan, Heni tidak bisa
memberikan saran apapun.

(E) Bersama para Camat di


Banyuwangi, rombongan tamu
dari Jepang itu mengikuti jamuan
makan malam.

2. Kalimat berikut yang ditulis


sesuai dengan aturan EYD
adalah

(A) Saya tidak akan membeli mobil


mahal, karena tidak punya uang.

(B) Oleh karena itu saya harus rajin


menabung.

(C) Atas bantuan saudara saya


mengucapkan terima kasih.

SOAL PENGANTAR 17 (D) Malam makin larut;


pekerjaannya belum selesai juga.
PUEBI 4
(E) Semua siswa baik yang laki-laki
maupun perempuan, mengikuti
ujian.
1. Kalimat berikut yang ditulis sesuai
dengan aturan EYD adalah

(A) Mereka menyepakati, bahwa 3. Pengendalian bahaya banjir yang


mulai tahun ajaran baru akan dilakukan pemerintah Indonesia
dilakukan kerja kelompok secara tidak akan berhasil kalau
lebih intensif. PERDA yang menaungi tidak
dibuat. Dengan demikian usaha
(B) Untuk mencapai target yang itu hanya akan membuang uang
diharapkan, perusahaan perlu sia-sia. Pemerintah perlu segera
melakukan peningkatan membuat PERDA khususnya
pengetahuan dan keterampilan yang menyangkut pembagian
karyawannya. tugas antarinstansi. Paragraf
tersebut akan menjadi paragraf
baku jika ejaannya diperbaiki dan jaringan yang cukup pesat,
sebagai berikut, KECUALI virus dilengkapi kemampuan
menyerang melalui jaringan
(A) huruf awal kata pemerintah komputer sehingga
ditulis huruf kapital. penyerangannya sangat cepat.

(B) kata PERDA ditulis dengan 5. Kalimat berikut yang seluruhnya


huruf kecil, kecuali huruf ditulis sesuai dengan aturan EYD
pertama. adalah

(C) sebelum kata usaha itu diberi (A) Kehematan yaitu salah satu ciri
tanda koma (,). kalimat efektif.

(D) sebelum kata khususnya diberi (B) Meskipun demikian, hal itu tidak
tanda koma (,). berarti bahwa mereka bebas dari
tugas.
(E) kata antarinstansi ditulis terpisah.
(C) Jika informasinya jelas kalimat
itu disebut kalimat efektif, karena
dapat dipahami isinya.
4. Kalimat berikut yang ditulis
sesuai dengan pedoman EYD, (D) Betapa pun panjangnya suatu
KECUALI kalimat yang penting adalah
informasinya.
(A) Pakistan dan Malaysia akan
bekerja sama melakukan riset, (E) Dalam kalimat efektif tidak ada
pengembangan, pelatihan, dan ide kalimat yang dapat
pendidikaan keuangan Islam. ditafsirkan secara ganda
maknanya.
(B) Tidak dapat dipungkiri bahwa
pendidikan multi kultural telah 6. Kalimat berikut yang dituli
meleburkan hegemoni pasar yang sesuai dengan EYD adalah
dikuasai lakilaki ke dalam
pangkuan kaum perempuan. (A) Tidak seorang bupati pun hadir
dalam acara halal bihalal di
(C) Bank pemerintah Pakistan telah Surabaya kemarin.
memberikan izin bagi enam bank
Islam dan sebelas bank (B) Hampir di setiap daerah, terdapat
konvensional untuk Universitas tetapi mahasiswanya
mengembangkan sistem sangat sedikit.
perbankan Islam.
(C) Dalam setiap Bab buku yang
(D) Saat ini, virus mempunyai varian berjudul Pernik-pernik
sasaran bukan hanya pada media Berbahasa, disajikan ringkas.
penyimpanan, melainkan pada
hampir semua hal yang (D) Selama menjadi direktur di Bank
berhubungan dengan komputer Araya, ia menunjukkan
dan sistem di dalamnya yang pengabdian yang baik
memungkinkan untuk diserang.
Dengan perkembangan internet
(E) Semua Fakultas Kedokteran di mudah mengikuti kuliah tentang
seluruh Indonesia saat ini Ekonomi Pembangunan.
melakukan akreditasi.
(C) Akuntansi terbagi dalam
beberapa bidang: akuntansi
sosial, akuntansi pemerintahan,
7. Krisis ekonomi, yang turut akuntansi perusahaan, dan
berperan dalam penambahan akuntansi organisasi nonprofit.
jumlah pengangguran di
Indonesia, juga ditambah (D) Diharapkan diktat yang berjudul
berbelitnya birokrasi di negara Akuntansi Sosial ini bermanfaat
ini, yang menjadikan para bagi para peminat akuntansi pada
investor asing enggan untuk umumnya dan mahasiswa
menanamkan modalnya ke sekolah tinggi ilmu ekonomi
Indonesia menyebabkan pada khususnya.
pemerintah harus berjuang
memperbaiki struktur (E) Penjelasan dengan asetilena
ekonominya. Kalimat tersebut dilakukan dengan cara membakar
akan menjadi baku bila bahan bakar gas asetilena dengan
diperbaiki dengan cara O2.

(A) menghilangkan tanda koma (,) 9. Beberapa analis asing maupun


sesudah kata ekonomi. domestik di pasar modal
berpendapat, kesimpangsiuran
(B) mengganti juga dengan kata susunan personalia kabinet
yang. merupakan faktor utama yang
menjadi penyebab indeks harga
(C) menghilangkan tanda koma (,) saham gabungan terpuruk.
sebelum kata juga. Sekalipun demikian, sentimen
negatif bursa regional yang
(D) menghilangkan untuk. berguguran turut memperparah
keadaan pasar saham. Akibatnya,
(E) menambah tanda koma (,) di indeks harga saham
depan kata menyebabkan. gabunganpun tidak mampu
mengatasi keterpurukannya.
Dalam paragraf di atas, terdapat
beberapa kesalahan ejaan dan
8. Penulisan kalimat berikut pemilihan kata. Paragraf tersebut
mengikuti aturan EYD, akan menjadi paragraf yang baik
kecuali .... jika direvisi dengan cara-cara
sebagai berikut, KECUALI ....
(A) Buku ini disusun untuk
membantu mahassiswa yang (A) gabunganpun (kalimat 3) ditulis
akan mengambil mata kuliah gabungan pun.
Akuntansi Perusahaan pada
fakultas ekonomi. (B) maupun (kalimat 1) diganti dan

(B) Kami menulis diktat ini dengan (C) Tanda koma (,) (kalimat 1)
maksud, agar mahasiswa lebih dihilangkan dan diganti bahwa.
(D) Sekalipun demikian (kalimat 2) (D) Menjelang operasi, pasien itu
diganti selain itu, berdoa kepada Tuhan Yang
Maha Esa.
(E) Analis (kalimat 1) diganti
analisis. (E) Secara kebetulan lokasi gedung
itu berada diantara perbatasan
10. Menulis merupakan ketrampilan dua wilayah.
yang paling sulit dikuasai. Dalam
proses menulis, banyak proses 12. Pemakaian tanda baca di bawah
pembelajaran pemakaian ejaan ini benar, kecuali
sesuai dengan kaidah bahasa
yang dianut, pembelajaran (A) Karena sering tidak masuk
penggunaan diksi maupun frase bekerja, pegawai itu mendapat
yang mengungkapkan pikiran. peringatan keras dari atasannya
Dalam kutipan di atas, terdapat
kesalahan tata tulis. Kesalahan (B) Ibu pergi ke toko membeli
yang dimaksud adalah sebagai barang-barang kebutuhan dapur
berikut, KECUALI : termos, gelas, dan kompor.

(A) kesalahan penulisan (C) Dalam tulisan tangan atau


diantaranya. ketikan, huruf atau kata yang
akan dicetak miring diberi satu
(B) kesalahan penggunaan tanda titik garis di bawahnya.
koma (;)
(D) Ia menyukai musik, teater dan
(C) kesalahan pemilihan kata jenis-jenis kesenian lainnya.
maupun.
(E) Laki-laki itu pintar, tetapi
(D) kesalahan penggunaan tanda sombong.
koma (,) sebelum kata hubung
dan.

(E) kesalahan penulisan kata


ketrampilan.
13. Pemakaian tanda baca yang
11. Diantara kalimat-kalimat berikut benar terdapat pada kalimat ....
yang benar ejaannya adalah
(A) Sahlan, mengucapkan terima
(A) Pelantikan para guru di kasih atas bantuan Putut.
selenggarakan di gedung serba
guna. (B) Semua peserta, yang tidak
membawa kartu peserta, harus
(B) Kemarin, Gubernur Yogyakarta melaporkan diri ke panitia.
meresmikan taman wisata di
Sleman. (C) Ratulangi tinggal di Jalan Mawar
5, Pati, Jawa Tengah.
(C) Jarak museum itu dari pusat kota
kurang lebih 1 km. (D) Ia harus pulang sekarang, karena
sakit.
(E) Sahabat saya Amin tinggal di anggaran belanja yang sangat
Manokwari. besar dibidang pendidikan. Kata
yang tepat untuk mengisi bagian
14. Penggunaan tanda baca yang kosong pada kalimat di atas
mengikutu EYD terdapat dalam adalah ....
kalimat ....
(A) memanipulasi
(A) Penggunaan obat tradisional,
termasuk jamu yang makin (B) mengakomodasi
marak dewasa ini adalah
kenyataan yang patut disyukuri. (C) menginvestasikan

(B) Kendati pemakaian jamu cukup (D) mengasuransikan


marak dan industri jamu juga
berkembang, sungguh (E) Mengalkulasi
mengherankan kalau ternyata
produksi tanaman obat beberapa
tahun belakangan stagnan.

(C) Dalam catatan H. Sampurno,


M.B.A. Kepala Badan POM
Indonesia memiliki 30.000 jenis
tumbuhan.

(D) Penurunan angka produksi


menjadi ironi, kalau kita
bandingkan dengan pasar yang
sesungguhnya membentang luas.

(E) Yang cukup menggembirakan,


adalah kenyataan bahwa
beberapa rumah sakit sudah
memasukkan obat tradisional
dalam terapi kepada pasien.

15. Pendidikan merupakan proses


pendewasaan bangsa yang akan
menjadi modal utama SOAL PENGANTAR 18
pembangunan. karena itu,
pemerintah Indonesia .... PUEBI 5
1. Penulisan kata gabung dibawah 3. Penulisan kata bilangan yang
ini benar ,KECUALI benar terdapat pada kalimat ....
(A) Dua ratus empat puluh orang
(A) Terminal bus antarkota di tamu diundang dalam pertemuan
Semarang selalu dilanda banjir itu.
(B) Dewi membaca novel itu hingga
(B) seorang residivis harus empat kali.
mempertanggungjawabkan (C) 14 orang tewas dalam kecelakaan
perbuatan kriminalnya. itu.
(D) Perusahaan itu mencari 10
(C) Rendy meneruskan studinya di (sepuluh) pegawai baru.
program pascasarjana Undip (E) Untuk membeli keperluan itu,
kami harus menyiapkan uang
(D) gerakan KB sebagai suatu 1000 an.
gerakan untuk mewujudkan
program caturwarga 4. Pemakaian tanda garis miring (/)
berikut ini sesuai PU EBI ,
(E) Karena prestasi akademiknya kecuali...
yang gemilang,Anton ditawari (A) Tahun akademik 2010/2011
bekerja di kantor asuransi akan segera kita masuki.
Bumiputera. (B) Acara akan berlangsung dari
13.00 s/d 14.30 WIB
2. Penulisan bilangan dalam (C) Nomor surat keterangan tersebut
kalimat yang sesuai dengan 21/PFL/XI/2008
aturan EYD adalah (D) Honornya Rp1.000.000,00/hari
(E) Jalan Kesuma IV/18
(A) Anda dapat membeli buku ini
seharga Rp50.000,- ditoko saya.
Teks untuk menjawab soal nomor 5-7!
(B) Sebanyak Sembilan puluh enam
kelurahan di Jakarta rawan (1)Pengelolaan sampah adalah
demam berdarah dengue (DBD) pengumpulan, pengangkutan,
pemrosesan, daur ulang, atau
(C) Pinjaman asosiasi pembangunan pembuangan dari material sampah.
internasional biasanya diberikan (2) Biasanya mengacu pada material
dengan jangka waktu tiga lima sampah yang dihasilkan dari kegiatan
tahun, masa tenggang sepuluh manusia, dan biasanya dikelola untuk
tahun, dan tingkat suku bunga mengurangi dampaknya terhadap
tetap 0,75% per tahun. kesehatan, lingkungan, atau estetika.
(3)Pengelolaan sampah juga dilakukan
(D) Ratusan orang yang berunjuk untuk memulihkan sumber daya alam
rasa mengaku perwakilan warga (resources recovery). (4)Pengelolaan [...]
Dayak dari 13 kabupaten dan bisa melibatkan zat padat, cair, gas, atau
kota di Kalimantan Timur radioaktif dengan metode dan
keterampilan khusus untuk masing-
(E) Harga kedelai impor naik sejak masing jenis zat. (5)Salah satu metode
pekan lalu dari Rp6.500 per yang digunakan adalah dengan cara
kilogram menjadi Rp7.000 penimbunan metode ini adalah metode
paling populer di dunia. (6) [...] ini panjang 8-10 cm, biasanya dihisap
biasanya dilakukan di tanah yang tidak seseorang setelah dibakar ujungnya.
terpakai lubang bekas pertambangan (2)Rokok merupakan pabrik bahan
atau lubang-lubang dalam (7)Sebuah kimia berbahaya. (3)Hanya dengan
lahan penimbunan darat yang dirancang membakar dan menghisap sebatang
dan dikelola dengan baik akan menjadi rokok saja, dapat diproduksi lebih dari
tempat penimbunan sampah yang 4000 jenis bahan kimia. (4)400
higienis dan murah. (8)Sedangkan diantaranya beracun dan 40 diantaranya
penimbunan darat yang tidak dirancang bisa berakumulasi dalam tubuh dan
dan tidak dikelola dengan baik akan dapat menyebabkan kanker. (5)Dengan
menyebabkan berbagai masalah kata lain, rokok termasuk golongan
lingkungan, di antaranya angin yang NAPZA (Narkotika, Psikotropika,
berbau sampah, menarik berkumpulnya Alkohol, dan Zat Adiktif
hama, dan adanya genangan air sampah.
(9) [...] samping lain dari sampah adalah (6)Merokok dapat
gas metana dan karbon dioksida yang membahayakan diri sendiri maupun
juga sangat berbahaya. orang disekitar. (7)Bahaya merokok
sudah terbukti menyebabkan berbagai
penyakit kronis seperti jantung koroner,
5. Kesalahan ejaan pada bacaan di kanker paru, penyakit paru obstruktif
atas terdapat pada .... dan stroke. (8)Pada kenyataannya,
(A) Kalimat ke (2) penyakit-penyakit tersebut baru sebagian
(B) Kalimat ke (5) dari bahaya merokok bagi kesehatan.
(C) Kalimat ke (4) (9)Pasalnya, ada banyak bahaya
(D) Kalimat ke (3) merokok lainnya yang tidak disadari
(E) Kalimat ke (1) seorang perokok. (10)Misalnya
penurunan daya tahan tubuh sehingga
6. Kesalahan tanda baca pada mereka lebih rentan terhadap penyakit
bacaan di atas terdapat pada.... infeksi. (11)Hal tersebut cenderung
(A) kalimat ke (1) terjadi secara singkat dan mungkin
(B) kalimat ke (2) menyebabkan dampak kesehatan pada
(C) kalimat ke (3) kehidupan sehari-hari seorang perokok.
(D) kalimat ke (4) (12)Rokok juga termasuk zat adiktif
(E) kalimat ke (6) karena dapat menyebabkan adiksi
(ketagihan) dan dependensi
7. Kata yang tepat untuk (ketergantungan) bagi orang yang
melengkapi bagian rumpang menghisapnya.
pada bacaan di atas adalah....
(A) kotoran, menimbun, efek 8.Kata yang tidak tepat pada bacaan
(B) sampah, penimbunan, efek di atas ialah....
(C) plastik, penimbunan, efek (A) "misalnya" pada kalimat (10)
(D) sampah, menimbun, akibat (B) "diantaranya" pada kalimat (4)
(E) daur ulang, penimbunan, efek (C) "pasalnya" pada kalimat (9)
(D) "dengan kata lain" pada kalimat
Teks untuk menjawab soal nomor 8-9! (5)
(E) "biasanya" pada kalimat (1)
(1)Rokok adalah lintingan atau
gulungan tembakau yang digulung /
dibungkus dengan kertas, daun, atau
kulit jagung, sebesar kelingking dengan
Tulisan berikut diikuti dua butir
9.Kesalahan tanda baca pada teks di pertanyaan. Pertimbangkan apakah kata
atas terdapat dalam kalimat.... atau kalimat pada setiap nomor bercetak
(A) (4) tebal TIDAK PERLU DIPERBAIKI (A)
(B) (2) atau diganti dengan pilihan lain yang
(C) (10) tersedia (B, C, D, atau E)
(D) (1)
(E) (5) Harga karet alam di pasar dunia
(13)Tahun 2013 tetap bagus. Seiring
meningkatnya produksi kendaraan,
permintaan terhadap karet meningkat.
10. Manakah penulisan singkatan Menurut (14)Chairman gabungan
dan akronim yang penulisannya Perusahaan Karet Indonesia, Asril Sutan
tepat? Amir, tingginya permintaan mendorong
peningkatan laju ekspor karet. Ekspor
(A) UUD, Pemilu, s/d karer tahun 2013 diperkirakan
meningkat (15)enam sampai dengan
(B) UPI, Unpad, Osis delapan persen dari total ekspor 2012
atau mencapai 2,4 juta ton. Ekspor karet
(C) Puskesmas, a.n. , Rp. tahun 2012 hingga oktober adalah
sebanyak 2,01 juta ton. Hingga akhir
(D) LIPI, Unpad, tilang 2012 ekspor karet bertambah 400.000
ton.
(E) OSIS, Ikapi (Ikatan Penerbit
Indonesia), daring (dalam jaringan)
13. (A) TIDAK PERLU
DIPERBAIKI
(B) tahun 2013
(C) “tahun 2013”
12. Manakah kelompok kata yang (D) tahun 2013
sesuai dengan kaidah PUEBI ? (E) “Tahun 2013”

(A) sukarela, antar warga 14. (A) TIDAK PERLU


DIPERBAIKI
(B) mempunyai, memerhatikan (B) ketua
(C) Ketua
(C) dukacita, semi final (D) Chairman
(E) “chairman”
(D) antar-jemput, tuna asmara

(E) non-Indonesia, saputangan


15. A) TIDAK PERLU
DIPERBAIKI
(B) 6-8 persen
(C) enam sd delapan persen
(D) enam sd delapan %
(E) enam sampai dengan delapan
persen
(A) Dengan teknik tersebut para
astronom dapat memperkirakan
SOAL PENGANTAR 19 ukuran planet meskipun belum
diketahui massanya, namun
GEJALA BAHASA & KALIMAT diperkirakan antara 5,7 hingga 11
EFEKTIF massa bumi.

(B) Landasan pacu Bandara Ahmad


Yani Semarang tergenang
1. Yang merupakan kalimat baku sepanjang 500 meter dan harus
adalah menggunakan 23 pompa air
untuk menyurutkan genangan.
(A) Buku ini terdapat istilah-istilah
kekerabatan pada orang Jawa dan (C) Bandara menunda semua
Batak dalam susunan masyarakat penerbangan sejak Minggu pagi
yang ditulis berdasarkan ilmu hingga pukul 21.00 karena
antropologi. kondisi landasan pacu masih
buruk.
(B) Dari beberapa pokok persoalan
yang diberikan untuk (D) Komisi pemilihan umum
membandingkan dua atau lebih berencana mengatur penetapan
dialek, antara lain dalam bidang calon terpilih yang berpihak pada
fonetik atau semantik. calon perempuan, jika partai
politik meraih tiga kursi
(C) Adapun yang akan saya uraikan DPR/DPRD, salah satunya mesti
di sini ialah kebersihan dan diberikan kepada calon
kesehatan, saya terdorong untuk perempuan yang perolehan
mengemukakannya, karena suaranya terbanyak.
Bering dilalaikan orang dan
sesungguhnya kebersihan dan (E) Berdasarkan pada data di Dinas
kesehatan itu perlu, sebab dengan Bina Marga Pemkot Surabaya,
bersih tentu akan menjadi sehat. maka titik-titik genangan itu
menyebar hampir di seluruh
(D) Karena nilai yang didapatkan wilayah Surabaya mulai sisi
lebih besar dari penolakan maka paling barat, timur, hingga ke
hipotesis nihil ditolak. selatan.

(E) Jika data yang diolah tidak 3. Kalimat yang merupakan kalimat
memenuhi syarat, balk dari segi baku adalah
kuantitas maupun kualitasnya,
hasil penelitian di lapangan itu (A) Ternyata jamu tidak hanya
tidak dapat terkenal di Indonesia saja, tetapi
dipertanggungjawabkan secara di Negara- Negara lain pun
ilmiah. mengkosumsi jamu

(B) Untuk Negara Asia di antaranya


ada Cina dan Thailand,
2. Kalimat berikut tidak baku, sedangkan untuk benua Eropa,
kecuali jamu banyak ditemukan di Eropa
Barat dan Jerman
(C) Sedangkan omzet produksi saat ini, mungkin sudah sangat
industri jamu Indonesia di dunia jauh berkurang.
berjumlah 300 juta dolar AS

(D) Penurunan nilai ekspor terjadi


karena adanya isu bahwa pada
obat-obatan herbal terdapat
kandungan bahan kimia
berbahaya

5. Kalimat yang tidak baku adalah


(E) Dan masalah kwalitas jamu yang
mengandung campuran kimia (A) Sesuai dangan kesepakatan,
kini dipermasalhkan oleh Negara pertunjukan itu akan diadakan di
importir karena ada yang Jawa dan Sumatra.
meninggal setelah
mengkonsumsi jamu tersebut. (B) Masa yang akan datang
4. Kalimat berikut ini yang kenaikan yang tidak terkendali
termasuk kalimat baku adalah dari harga kebutuhan sehari-hari
(A) Sarana dan prasarana yang tidak boleh terjadi lagi.
disediakan oleh pihak sekolah itu
untuk memfasilitasi siswa yang (C) Ramalan BMG semakin kita
aktif dan kreatif agar dapat perhatikan, terutama setelah
muncul berbagai dampak tak
mengembangkan minat bakat
terduka dari perubahan iklim.
mereka
(B) Pada minggu kedua pertemuan (D) Di negara-negara bermusim
kita akan membicarakan tentang empat, ramalan cuaca dan
pola-pola sinkretisme dan geofisika sudah lama menjadi
akulturasi budaya yang terjadi di bagian dari kehidupan sehari-
nusantara. hari.
(C) Jika disampaikan secara lebih
(E) Untuk keperluan pelestarian
sistematis dengan ilustrasi yang lingkungan diperlukan
memadai, maka brosur ini akan tanggungjawab kolektif dan
memberikan sugesti yang lebih antisipatif dari para pengusaha
kuat pada pembacanya. yang bergerak di bidang
(D) Sebelum presiden memberikan pengusahaan hutan.
kesempatan kepada hadirin untuk
6. (1) Burung termasuk dalam jenis
bertanya, maka terlebih dahulu
unggas yang bisa terbang . (2)
meminta seorang stafnya Namun, ada burung yang tidak
memandu jalanya Temu Wicara. bisa terbang. (3) Contohnya
(E) Kalau dibandingkan dengan burung untuk (struthio camelus).
situasi politik masa lalu, praktik (4) Burung unta adalah burung
KKN, yakni kecenderungan terbesar yang masih hidup dan
tidak bias terbang (5) Dahulu,
perilaku korupsi, kolusi, dan
burung ini terdapat dalam jumlah
nepotisme oleh jajaran birokrasi, besar di Afrika dan Asia Barat
Daya. (6) Sekarang , burungnya pengajaran tahun depan akan di
ini hanya ditemukan di beberapa ikuti oleh mahasiswa idealis
daerah di Asia Timur. (7) Ada yang memiliki visi dan misi
juga burung unta yang hidup di serta imajinasi bagus itu.
peternakan di Afrika Selatan. (8)
Sementara itu, di Australia (C) . Kalimat di atas sama idenya
Selatan, ada burung unta yang dengan kalimat Tahun depan
sudah dijadikan hewan piaraan. program pertukaran mahasiswa
(9) Seperti halnya burung lain Indonesia-Jepang akan
yang tidak bisa terbang, burung diikutinya.
unta sudah terbiasa pada
kehidupan di tanah dan dapat (D) Kalimat di atas tidak dapat
berlari dengan baik. (10) Dalam diubah menjadi kalimat pasif.
hal ini ayam tidak bisa
disamakan dengan burung unta. (E) Dengan arti atau maksud yang
Paragraf di atas akan merupakan sama, kalimat diatas dapat
paragraf yang memiliki kepaduan diubah menjadi Mahasiswa itu
jika…. tahun depan akan mengikuti
program pertukaran mahasiswa
(A) Kalimat (2) dan (3) digabungkan. Indonesia-Jepang karena
memiliki visi dan misiserta
(B) Kalimat (8) dan (9) dihilangakn. imajinasi bagus.

(C) Kalimat (10) dihilangkan 8. Pembentukan kata serapan yang


betul terdapat dalam kalimat
(D) Kalimat (7) dihilangkan
(A) Di dunia olahraga yang di
(E) Kalimat (7), (8) dan (9) pentingkan adalah sportivitas.
digabungkan.
(B) Sikap yang kaku dari seorang
7. Mahasiswa idealis yang memiliki pemimpin menimbulkan watak
visi dan misi serta imajinasi arogan dan otoritair.
bagus itu tahun depan akan
mengikuti program pertukaran (C) Majelis juga mengusulkan, agar
mahasiswa Indonesia – Jepang kalangan eksekutip dan legislatip
yang merupakan realisasi kerja peka terhadap aspirasi rakyat.
sama bidang pendidikan dan
pengajaran. Pernyataan yang (D) Mereka juga memprotes
benar berkaitan dengan kwalitas beras yang mereka
kalimat… terima.

(A) Yang menduduki fungsi subjek (E) Rapat itu gagal karena yang hadir
dalam kalimat di atas adalah tidak memenuhi korum.
mahasiswa.

(B) Kalimat di atas dapat diubah


menjadi Program pertukaran 9. Kalimat yang menggunakan
mahasiswa Indonesia – Jepang ragam baku adalah…
yang merupakan realisasi kerja
sama bidang pendidikan dan
(A) Dari hasil penelitian (E) E. Kerjasama Indonesia- Timur
membuktikan bahwa epilepsi Tengah yang digalakkan oleh
bukan penyakit menular. kedua belah pihak.

(B) Mengenai rancangan undang –


undang itu belum disetujui
anggota DPR. 11. Di antara kalimat berikut ini
yang menggunakan ragam
(C) Hari ini Indonesia akan kalimat baku adalah
memberangkatkan tiga belas
mahasiswa untuk mengikuti (A) Garis keturunan dari pihak ibu
olimpiade matematika di masih sulit diakomodasi dalam
Perancis. RUU Kewarganegaraan

(D) Menurut penelitian Depbudpar (B) Menginjak tahun 1970-an


2005 mengungkapkan orang kemajuan ilmu dan tekonologi di
Indonesia yang berlibur ke luar segala bidang maju pesat dengan
negeri rata-rata mengeluarkan ditemukannya microchip di mana
dana 860 dollar AS dalam setiap merupakan komponen dasar
kunjungan. komputer

(E) Karya sastra melukiskan hakikat (C) Dari dialog itu tampak banyak
yang nyata dari objek sebagai sekali kelemahan daripada
titik tolak, kemudian dengan Undang-undang Nomor 62 itu
perlahan bergerak ke arah
imajinasi dan perasaan. (D) . Ketidakadilan dalam RUU itu
terlihat terutama dalam ketentuan
yang mana mengatur perkawinan
antar-bangsa

(E) Banyak pengusaha nakal yang


10. Deretan kata yang melarikan diri ke luar negeri
mengemukakan ide yang utuh dengan membawa asset dari
adalah… negara secara melanggar hukum

(A) Perubahan sistem seleksi


mahasiswa baru khususnya di
UGM.

(B) Penegakan keadilan yang harus 12. Dalam pasal 9 ayat 2 RUU
didukung segenap elemen Bahasa menyebutkan bahwa
bangsa. pidato kenegaraan yang
disampaikan baik di dalam
(C) Memudarnya semangat gotong- maupun di luar negeri harus
royong yang perlu diperhatikan menggunakan Bahasa Indonesia
yang baik dan benar Agar
(D) Tingginya usia harapan hidup kalimat tersebut baku, kata yang
merupakan salah satu indikator perlu dihilangkan adalah
makin baiknya tingkat
kesejahteraan masyarakat. (A) dalam (sebelum kata pasal)
(B) menyebutkan Mongoloid dan
Australomelanesid
(C) bahwa
(B) Dari karakter fisik manusia
(D) harus Indonesia ditunjukkan bahwa
warna kulit, lipatan kelopak
(E) dalam (sesudah kata depan) mata, dan rambut yang secara
umum digolongkan ke dalam ras
13. …panen padi belum merata, Mongoloid dan
sejumlah pengumpul gabah Australomelanesid
mengaku kesulitan mencari stok
gabah kering panen…ada, (C) Karakter fisik manusia Indonesia
rendemen hasil panen padi lokal menunjukkan bahwa warna kulit,
jauh lebih rendah…biasanya. lipatan kelopak mata, dan rambut
Kelangkaan stok gabah kering yang secara umum digolongkan
panen…disebabkan tidak adanya ke dalam ras Mongoloid dan
panen,…berubahnya sistem Australomelanesid
pemesanan gabah. Kata
penghubung yang tepat untuk (D) Karakter fisik manusia Indonesia
mengisi titik-titik di atas adalah ditunjukkan oleh warna kulit,
lipatan kelopak mata, dan rambut
(A) karena, jika, karena, dan yang secara umum digolongkan
ke dalam ras Mongoloid dan
(B) sebab, ketika, jika, makna Australomelanesid

(C) akibat, kalaupun, daripada, (E) Karakter fisik manusia Indonesia


bukan, melainkan menunjukkan bahwa warna kulit,
lipatan kelopak mata, dan rambut
(D) karena, ketika, dari, tidak, tetapi secara umum digolongkan ke
dalam ras Mongoloid dan
(E) akibat, ketika, dan, kalau, maka Australomelanesid

14. Dari karakter fisik manusia


Indonesia menunjukkan warna
kulit. Lipatan kelopak mata,
rambut yang secara umum
digolongkan ke dalam ras
Mongoloid dan
Australomelanesid Kalimat
tersebut menjadi baku jika
diubah menjadi .....

(A) Dari karakter fisik manusia


Indonesia menunjukkan warna
kulit, lipatan kelopak mata, dan
rambut yang secara umum
digolongkan ke dalam ras
15. Gerak daratan itu tidak akan
pernah usai sepanjang usia bumi
ini. Kalimat ini mengandung
maksud bahwa ....

(A) usia bumi ini akan berakhir


ketika daratan di permukaan
bumi berhenti bergerak.

(B) daratan di muka bumi bergerak


sepanjang luas permukaan bumi.

(C) gerak daratan di muka bumi akan


berakhir setelah mengelilingi
seluruh permukaan bumi

(D) usia bumi ini akan bertambah


panjang apabila daratan di
permukaannya masih terus
bergerak.

(E) daratan di muka bumi akan terus


bergerak selama bumi ini masih
ada.

Anda mungkin juga menyukai